NCLEX and Pharm Review - Exam 3

Lakukan tugas rumah & ujian kamu dengan baik sekarang menggunakan Quizwiz!

A client tells the nurse about feeling pressure to spend every Sunday with family. However, the client's spouse does not want to participate and stays at home waiting for the client to return. Which is determining this client's self-concept? a. Family and culture b. Resources c. History of successes and failures d. Stressors

a

A client worries every day about personal health and states, "I may not have enough medication if the weather takes a turn for the worse." Based on this data, which diagnosis does the nurse anticipate for this client? a. Generalized anxiety disorder b. Obsessive-compulsive disorder c. Phobia d. Panic disorder

a

A person with hypertension and known heart disease has frequent bronchospasms and asthma attacks that are most responsive to sympathomimetic drugs. This patient might be best treated with a. an inhaled sympathomimetic to decrease systemic effects. b. a xanthine. c. no sympathomimetics because they would be contraindicated. d. an anticholinergic.

a

Antihistamines should be used cautiously in patients with a. histories of arrhythmias or prolonged QT intervals. b. COPD or bronchitis. c. asthma or seasonal rhinitis. d. angioedema or low blood pressure.

a

Barbiturates cause liver enzyme induction, which could lead to a. rapid metabolism and loss of effectiveness of other drugs metabolized by those enzymes. b. increased bile production. c. CNS depression. d. the need to periodically lower the barbiturate dose to avoid toxicity.

a

Clinically, dysrhythmias, or arrhythmias, may cause a. altered cardiac output that could affect all cells. b. changes in capillary filling pressures. c. alterations in osmotic pressure. d. valvular dysfunction.

a

Coronary artery disease results in a. an imbalance in cardiac muscle oxygen supply and demand. b. delivery of blood to the heart muscle during systole. c. increased pulse pressure. d. a decreased workload on the heart.

a

Diffusion of CO2 from the tissues into the capillary blood a. occurs if the tissue concentration of CO2 is greater than that in the blood. b. decreases as blood acidity increases. c. increases in the absence of carbonic anhydrase. d. is accompanied by a decrease in plasma bicarbonate.

a

Drugs that are best used to cause a patient to sleep are called a. hypnotics. b. sedatives. c. antiepileptics. d. anxiolytics.

a

Local anesthetics are used to block feeling in specific body areas. If given in increasing concentrations, local anesthetics can cause loss in the following order: a. Temperature sensation, touch sensation, proprioception, and skeletal muscle tone b. Touch sensation, skeletal muscle tone, temperature sensation, and proprioception c. Proprioception, skeletal muscle tone, touch sensation, and temperature sensation d. Skeletal muscle tone, touch sensation, temperature sensation, and proprioception

a

The benzodiazepines are the most frequently used anxiolytic drugs because a. they are anxiolytic at doses much lower than those needed for sedation or hypnosis. b. they can also be stimulating. c. they are more likely to cause physical dependence than older anxiolytic drugs. d. they do not affect any neurotransmitters.

a

The benzodiazepines react with a. GABA receptor sites in the RAS to cause inhibition of neural arousal. b. norepinephrine receptor sites in the sympathetic nervous system. c. acetylcholine receptor sites in the parasympathetic nervous system. d. monoamine oxidase to increase norepinephrine breakdown.

a

The nurse is concerned that a client with an alteration in perfusion is at risk for inadequate oxygenation. What should the nurse consider when planning for this client's potential health problem? a. Instructing on deep breathing b. Positioning to increase blood return c. Encouraging ambulation every thirty minutes d. Administering medications appropriate to increase heart rate

a

Treatment of obstructive pulmonary disorders is aimed at a. opening the conducting airways or decreasing the effects of inflammation. b. blocking the autonomic reflexes that alter respirations. c. blocking the effects of the immune and inflammatory systems. d. altering the respiratory membrane to increase the flow of oxygen and carbon dioxide.

a

Which is correct regarding the cardiac cycle? a. Blood that has a high amount of oxygen enters the left atrium. b. Blood that has a high amount of oxygen enters the right ventricle. c. The highest pressure is found in the right ventricle of the heart. d. The mitral valve allows for bidirectional blood flow.

a

Friends of a client hospitalized with asthma would like to bring the client a gift. Which gift would the nurse recommend for this client? a. A book b. Fruit and candy c. A stuffed animal d. A basket of flowers

a A client with asthma must not be exposed to items that can exacerbate their disease process. Specific allergens, chemicals, and foods must be avoided. Flowers, food, and items that may contain dust and much, such as a stuffed animal, should be avoided. Objects void of irritants, such as a book, would be an appropriate gift.

A client states to the nurse, "I know I have high blood pressure but I don't want to take medication." Based on this data, which health problem is the client at risk for developing? a. Cardiomyopathy b. Metabolic syndrome c. Diabetes d. Gastritis

a Hypertension places the client at risk for development of cardiomyopathy. Hypertension has not been associated with metabolic syndrome, diabetes, or gastritis.

A client is complaining of frequent headaches, chest tightness, palpitations, and menstrual irregularities. The client also reports having lost weight and experiencing difficulty eating and sleeping. The nurse notes that the client is tearful, sad, and lacks energy. Which question is most appropriate when assessing the source of the client's symptoms? a. "Have you experienced a loss of a loved one recently?" b. "Why are you crying so much?" c. "Can you tell me why you are so sad?" d. "How long have you been grieving?"

a The client is exhibiting the classic signs of grief and possibly unresolved grief, which affects the physical health of the client. Asking the client why she is sad does encourage her to express her feelings, but in this case the nurse needs to know the type of grief the client is experiencing and how long the client has been in this state in order to plan appropriate care. Asking the client why she is crying is a challenge, as is asking the client how long she has been grieving. The nurse should be supportive and nonjudgmental.

While helping a client with the evening meal, the nurse observes the client close his eyes, bow his head, and murmur words of thanks and praise. What does this behavior indicate to the nurse? a. The client was praying before eating. b. The client did not want the nurse to leave. c. The client is confused d. The client was asking that the meal be better than the last.

a The client's behavior of bowing the head, closing the eyes, and murmuring words of thanks and praise are indications that the client was praying. The client was not demonstrating confusion. The nurse has no way of knowing if the client was asking that the meal be better than the last. The client was not delaying the nurse so that she did not leave.

The postoperative nurse is planning care for a client recovering from major thoracic surgery. Which nursing diagnoses should the nurse select to plan for this client's immediate care needs? Select all that apply. a. Risk for Impaired Gas Exchange b. Risk for Imbalanced Nutrition: Less than Body Requirements c. Risk for Decreased Cardiac Output d. Risk for Imbalanced Fluid Volume e. Risk for Ineffective Airway Clearance

a c d

Hageman factor is known to activate which? Select all that apply. a. The clotting cascade b. The anticlotting process c. The inflammatory response d. Platelet aggregation e. Thromboxane A2 f. Troponin coupling

a, b, c

What would a nurse expect to assess if a patient has inflammation of the upper respiratory tract? Select all that apply. a. A runny nose b. Laryngitis c. Sneezing d. Hypoxia e. Rales f. Wheezing

a, b, c

Common adverse effects associated with the use of topical nasal steroids would include which conditions? Select all that apply. a. Local burning and stinging b. Dryness of the mucosa c. Headache d. Constipation and urinary retention e. Fungal infections f. Osteonecrosis

a, b, c, e

In assessing a client who is experiencing anxiety, the nurse would expect to find which conditions? Select all that apply. a. Rapid breathing b. Rapid heart rate c. Fear and apprehension d. Constricted pupils e. Decreased abdominal sounds f. Hypotension

a, b, c, e

During diastole, which would occur? Select all that apply. a. Opening of the AV valves b. Relaxation of the myocardial muscle c. Flow of blood from the atria to the ventricles d. Contraction of the ventricles e. Closing of the semilunar valves f. Filling of the coronary arteries

a, b, c, e, f

Evaluating a client who is taking an anticoagulant for blood loss would usually include assessing for which conditions? Select all that apply. a. The presence of petechiae b. Bleeding gums while brushing the teeth c. Dark-colored urine d. Yellow color to the sclera or skin e. The presence of ecchymotic areas f. Loss of hair

a, b, c, e, f

An acute myocardial infarction is usually associated with which conditions? Select all that apply. a. Permanent injury to the heart muscle b. Potentially serious arrhythmias c. Pain d. The development of hypertension e. Loss of consciousness f. A feeling of anxiety

a, b, c, f

Antiplatelet drugs block the aggregation of platelets and keep vessels open. These drugs would be useful in which circumstances? Select all that apply. a. Maintaining the patency of grafts b. Decreasing the risk of fatal MI c. Preventing reinfarction after MI d. Dissolving a PE and improving oxygenation e. Decreasing damage in a subarachnoid bleed f. Preventing thromboembolic strokes

a, b, c, f

Balanced anesthesia combines different classes of drugs to achieve the best effects with the fewest adverse effects. Balanced anesthesia usually involves the use of which types of drugs? Select all that apply. a. Anticholinergics b. Narcotics c. Sedative/hypnotics d. Adrenergic beta-blockers e. Dantrolene f. Neuromuscular blocking agents

a, b, c, f

The nose performs which functions in the respiratory system? Select all that apply. a. Serves as a passageway for air movement b. Warms and humidifies the air c. Cleanses the air using hair fibers d. Stimulates surfactant release from the alveoli e. Initiates the cough reflex f. Initiates the sneeze reflex

a, b, c, f

The sympathetic nervous system would be expected to have which effects? Select all that apply. a. Stimulates the heart to beat faster b. Speeds conduction through the AV node c. Causes the heart muscle to contract harder d. Slows conduction through the AV node e. Decreases overall vascular volume f. Increases total peripheral resistance

a, b, c, f

Your client has a long history of anxiety and has always responded well to diazepam. She has just learned that she is pregnant and feels very anxious. She would like a prescription for diazepam to get her through her early anxiety. What rationale would the nurse use in explaining why this is not recommended? Select all that apply. a. This drug is known to cause a predictable syndrome of birth defects, including cleft lip and pyloric stenosis. b. Babies born to mothers taking benzodiazepines may progress through a neonatal withdrawal syndrome. c. Cardiac defects and small brain development may occur if this drug is taken in the first trimester. d. This drug almost always causes loss of the pregnancy. e. The hormones the body produces during pregnancy will make you unresponsive to diazepam. f. This drug could have adverse effects on your baby; we should explore nondrug measures to help you deal with the anxiety.

a, b, c, f

A nurse is preparing a family for a terminal weaning of a loved one. Which nursing actions would facilitate this process? Select all that apply. a. Participate in the decision-making process by offering the family information about the advantages and disadvantages of continued ventilatory support. b. Explain to the family what will happen at each phase of the weaning and offer support. c. Check the orders for sedation and analgesia, making sure that the anticipated death is comfortable and dignified. d. Tell the family that death will occur almost immediately after the patient is removed from the ventilator. e. Tell the family that the decision for terminal weaning of a patient must be made by the primary care provider. f. Set up mandatory counseling sessions for the patient and family to assist them in making this end-of-life decision.

a, b, c. A nurse's role in terminal weaning is to participate in the decision-making process by offering helpful information about the benefits and burdens of continued ventilation and a description of what to expect if terminal weaning is initiated. Supporting the patient's family and managing sedation and analgesia are critical nursing responsibilities. In some cases, competent patients decide that they wish their ventilatory support ended; more often, the surrogate decision makers for an incompetent patient determine that continued ventilatory support is futile. Because there are no guarantees how any patient will respond once removed from a ventilator, and because it is possible for the patient to breathe on his or her own and live for hours, days, and, rarely, even weeks, the family should not be told that death will occur immediately. Counseling sessions may be arranged if requested but are not mandatory to make this decision.

A nurse is assessing patients in a skilled nursing facility for sleep deficits. Which patients would be considered at a higher risk for having sleep disturbances? Select all that apply. a. A patient who has uncontrolled hypothyroidism. b. A patient with coronary artery disease. c. A patient who has GERD. d. A patient who is HIV positive. e. A patient who is taking corticosteroids for arthritis. f. A patient with a urinary tract infection.

a, b, c. A patient who has uncontrolled hypothyroidism tends to have a decreased amount of NREM sleep, especially stages II and IV. The pain associated with coronary artery disease and myocardial infarction is more likely with REM sleep, and a patient who has GERD may awaken at night with heartburn pain. Being HIV positive, taking corticosteroids, and having a urinary tract infection does not usually change sleep patterns.

A nurse midwife is assisting a patient who is firmly committed to natural childbirth to deliver a full-term baby. A cesarean delivery becomes necessary when the fetus displays signs of distress. Inconsolable, the patient cries and calls herself a failure as a mother. The nurse notes that the patient is experiencing what type of loss? Select all that apply. a. Actual b. Perceived c. Psychological d. Anticipatory e. Physical f. Maturational

a, b, c. The losses experienced by the woman are actual, perceived, and psychological. Actual loss can be recognized by others as well as by the person sustaining the loss; perceived loss is experienced by the person but is intangible to others; and psychological loss is a loss that is felt mentally as opposed to physically. Anticipatory loss occurs when one grieves prior to the actual loss; physical loss is loss that is tangible and perceived by others; and maturational loss is experienced as a result of natural developmental processes.

A nurse would anticipate the use of general anesthetics for which reasons? Select all that apply. a. To produce analgesia b. To produce amnesia c. To activate the reticular activating system d. To block muscle reflexes e. To cause unconsciousness f. To prevent nausea

a, b, d, e

Pressure within the vascular system is determined by which? Select all that apply. a. Peripheral resistance b. Stroke volume c. Sodium load d. Heart rate e. Total intravascular volume f. Rate of erythropoietin release

a, b, d, e

The renin-angiotensin system is associated with which? Select all that apply. a. Intense vasoconstriction and BP elevation b. Blood flow through the kidneys c. Production of surfactant in the lungs d. Release of aldosterone from the adrenal cortex e. Retention of sodium and water in the kidneys f. Liver production of fibrinogen

a, b, d, e

When describing the action of antianginal drugs to a patient, which would the nurse include? Select all that apply. a. Decrease the workload on the heart b. Increase the supply of oxygen to the heart c. Change the metabolic pathway in the heart muscle to remove the need for oxygen d. Restore the supply-and-demand balance of oxygen in the heart e. Decrease venous return to the heart f. Alter the coronary artery filling pathway

a, b, d, e

Additional nursing interventions for clients receiving antihistamines would probably include which recommendations? Select all that apply. a. Using a humidifier b. Advising the client to suck sugarless lozenges to help relieve the dry mouth c. Limiting fluid intake to decrease swelling d. Providing safety measures to prevent falls or injury e. Encouraging pushing fluids, if allowed f. Leaving bowls of water around the house to increase humidity

a, b, d, e, f

Treating angina involves modifying factors that could decrease myocardial oxygen consumption. It could be expected that this might include... Select all that apply. a. weight loss. b. use of nitrates. c. use of angiotensin-converting enzyme inhibitors. d. activity modification. e. use of a piperazine acetamide agent. f. use of a calcium channel blocker.

a, b, d, e, f

Which assessments and interventions should the nurse consider when performing tracheal suctioning? Select all that apply. a. Closely assess the patient before, during, and after the procedure. b. Hyperoxygenate the patient before and after suctioning. c. Limit the application of suction to 20 to 30 seconds. d. Monitor the patient's pulse frequently to detect potential effects of hypoxia and stimulation of the vagus nerve. e. Use an appropriate suction pressure (80 to 150 mm Hg). f. Insert the suction catheter no further than 1 cm past the length of the tracheal or endotracheal tube.

a, b, d, e. Close assessment of the patient before, during, and after the procedure is necessary to limit negative effects. Risks include hypoxia, infection, tracheal tissue damage, dysrhythmias, and atelectasis. The nurse should hyperoxygenate the patient before and after suctioning and limit the application of suction to 10 to 20 seconds. The nurse should also take the patient's pulse frequently to detect potential effects of hypoxia and stimulation of the vagus nerve. Using an appropriate suction pressure (80 to 150 mm Hg) will help prevent atelectasis related to the use of high negative pressure. Research suggests that insertion of the suction catheter should be limited to a predetermined length (no further than 1 cm past the length of the tracheal or endotracheal tube) to avoid tracheal mucosal damage, including epithelial denudement, loss of cilia, edema, and fibrosis.

Which comfort measures are important for a patient receiving a local anesthetic? Select all that apply. a. Skin care and turning b. Reassurance over loss of control and sensation c. Use of antihypertensive agents d. Use of analgesics as needed e. Ice applied to the area involved f. Safety precautions to prevent injury

a, b, d, f

A nurse is caring for patients admitted to a long-term care facility. Which nursing actions are appropriate based on the religious beliefs of the individual patients? Select all that apply. a. The nurse dietitian asks a Buddhist if he has any diet restrictions related to the observance of holy days. b. A nurse asks a Christian Scientist who is in traction if she would like to try nonpharmacologic pain measures. c. A nurse administering medications to a Muslim patient avoids touching the patient's lips d. A nurse asks a Roman Catholic woman if she would like to attend the local Mass on Sunday. e. The nurse is careful not to schedule treatment and procedures on Saturday for a Hindu patient. f. The nurse consults with the medicine man of a Native American patient and incorporates his suggestions into the care plan.

a, b, d, f. The nurse dietitian should ask a Buddhist if he has any diet restrictions related to the observance of holy days. Since Catholic Scientists avoid the use of pain medications, the nurse should ask a Christian Scientist who is in traction if she would like to try nonpharmacologic pain measures. A nurse administering medications to a Hindu woman avoids touching the patient's lips. A nurse should ask a Roman Catholic woman if she would like to attend the local Mass on Sunday. The nurse is careful not to schedule treatment and procedures on Saturday for a Jewish patient due to observance of the Sabbath. The nurse would appropriately consult with the medicine man of a Native American patient and incorporates his or her suggestions into the care plan.

Teaching a client who is prescribed an HMG-CoA reductase inhibitor to treat high cholesterol and high lipid levels should include which information? Select all that apply. a. The importance of exercise b. The need for dietary changes to alter cholesterol levels c. That taking a statin will allow a full, unrestricted diet d. That drug therapy is always needed when these levels are elevated e. The importance of controlling blood pressure and blood glucose levels f. That stopping smoking may also help to lower lipid levels

a, b, e, f

A nurse is caring for an older adult who is having trouble getting to sleep at night and formulates the nursing diagnosis Disturbed sleep pattern: Initiation of sleep. Which nursing interventions would the nurse perform related to this diagnosis? Select all that apply. a. Arrange for assessment for depression and treatment. b. Discourage napping during the day. c. Decrease fluids during the evening. d. Administer diuretics in the morning. e. Encourage patient to engage in some type of physical activity. f. Assess medication for side effects of sleep pattern disturbances.

a, b, e, f. For patients who are having trouble initiating sleep, the nurse should arrange for assessment for depression and treatment, discourage napping, promote activity, and assess medications for sleep disturbance side effects. Limiting fluids and administering diuretics in the morning are appropriate interventions for Disturbed Sleep Pattern: Maintaining Sleep.

A certified nurse midwife is teaching a pregnant woman techniques to reduce the pain of childbirth. Which stress reduction activities would be most effective? Select all that apply. a. Progressive muscle relaxation b. Meditation c. Anticipatory socialization d. Biofeedback e. Rhythmic breathing f. Guided imagery

a, b, e, f. Relaxation techniques are useful in many situations, including childbirth, and consist of rhythmic breathing and progressive muscle relaxation. Meditation and guided imagery could also be used to distract a patient from the pain of childbirth. Anticipatory socialization helps to prepare people for roles they don't have yet, but aspire to, such as parenthood. Biofeedback is a method of gaining mental control of the autonomic nervous system and thus regulating body responses, such as blood pressure, heart rate, and headaches.

A nurse has been asked to witness a patient signature on an informed consent form for surgery. What information should be included on the form? Select all that apply. a. The option of nontreatment b. The underlying disease process and its natural course c. Notice that once the form is signed, the patient cannot withdraw the consent d. Explanation of the guaranteed outcome of the procedure or treatment e. Name and qualifications of the provider of the procedure or treatment f. Explanation of the risks and benefits of the procedure or treatment

a, b, e, f. The information contained in informed consent includes the description of the procedure or treatment, potential alternative therapies, and the option of nontreatment, the underlying disease process and its natural course, the name and qualifications of the health care provider performing the procedure or treatment, explanation of the risks and benefits, explanation that the patient has the right to refuse treatment and consent can be withdrawn, and explanation of expected (not guaranteed) outcome, recovery, and rehabilitation plan and course.

A nurse is teaching a patient with a sleep disorder how to keep a sleep diary. Which data would the nurse have the patient document? Select all that apply. a. Daily mental activities b. Daily physical activities c. Morning and evening body temperature d. Daily measurement of fluid intake and output e. Presence of anxiety or worries affecting sleep f. Morning and evening blood pressure readings

a, b, e. A sleep diary includes mental and physical activities performed during the day and the presence of any anxiety or worries the patient may be experiencing that affect sleep. A record of fluid intake and output, body temperature, and blood pressure is not usually kept in a sleep diary.

A nurse who cared for a dying patient and his family documents that the family is experiencing a period of mourning. Which behaviors would the nurse expect to see at this stage? Select all that apply. a. The family arranges for a funeral for their loved one. b. The family arranges for a memorial scholarship for their loved one. c. The coroner pronounces the patient's death. d. The family arranges for hospice for their loved one. e. The patient is diagnosed with terminal cancer. f. The patient's daughter writes a poem expressing her sorrow.

a, b, f. Mourning is defined as the period of acceptance of loss and grief, during which the person learns to deal with loss. It is the actions and expressions of that grief, including the symbols and ceremonies (e.g., a funeral or final celebration of life), that make up the outward expressions of grief. A diagnosis of cancer and the coroner's pronouncing the patient's death are not behaviors of the family during a period of mourning. Arranging for hospice care would not be an expression of mourning.

A nurse is assessing a patient who complains of migraines that have become "unbearable." The patient tells the nurse, "I just got laid off from my job last week and I have two kids in college. I don't know how I'm going to pay for it all." Which physiologic effects of stress would be expected findings in this patient? Select all that apply. a. Changes in appetite b. Changes in elimination patterns c. Decreased pulse and respirations d. Use of ineffective coping mechanisms e. Withdrawal f. Attention-seeking behaviors

a, b. Physiologic effects of stress include changes in appetite and elimination patterns as well as increased (not decreased) pulse and respirations. Using ineffective coping mechanisms, becoming withdrawn and isolated, and exhibiting attention-seeking behaviors are psychological effects of stress.

A scrub nurse is assisting a surgeon with a kidney transplant. What are the patient responsibilities of the scrub nurse? Select all that apply. a. Maintaining sterile technique b. Draping and handling instruments and supplies c. Identifying and assessing the patient on admission d. Integrating case management e. Preparing the skin at the surgical site f. Providing exposure of the operative area

a, b. The scrub nurse is a member of the sterile team who maintains sterile technique while draping and handling instruments and supplies. Two duties of the circulating nurse are to identify and assess the patient on admission to the OR and prepare the skin at the surgical site. The RNFA actively assists the surgeon by providing exposure of the operative area. The APRN coordinates care activities, collaborates with physicians and nurses in all phases of perioperative and postanesthesia care, and integrates case management, critical paths, and research into care of the surgical patient.

A client reports that he thinks he is taking an antidepressant, but he is not sure. In reviewing his medication history, which drugs would be considered antidepressants? Select all that apply. a. Tetracyclic drugs b. Cholinergics c. SSRIs d. MAOIs e. Angiotensin II receptor blockers f. Benzodiazepine

a, c, d

Depression is a common affective disorder that strikes many people. In assessing a client who might be suffering from depression, the nurse would expect to find which conditions? Select all that apply. a. Lack of energy b. Hyperactivity c. Sleep disturbances d. Libido problems e. Confusion f. Decreased reflexes

a, c, d

An antihistamine would be the drug of choice for treating which conditions? Select all that apply. a. Itchy eyes b. Irritating cough c. Nasal congestion d. Runny nose e. Idiopathic urticaria f. Thick, tenacious secretions

a, c, d, e

A child with repeated asthma attacks may be treated with which drugs? Select all that apply. a. A leukotriene receptor antagonist b. A beta-blocker c. An inhaled corticosteroid d. An inhaled beta-agonist e. A surfactant f. A mast cell stabilizer

a, c, d, f

A nurse witnesses a street robbery and is assessing a patient who is the victim. The patient has minor scrapes and bruises, and tells the nurse, "I've never been so scared in my life!" What other symptoms would the nurse expect to find related to the fight-or-flight response to stress? Select all that apply. a. Increased heart rate b. Decreased muscle strength c. Increased mental alertness d. Increased blood glucose levels e. Decreased cardiac output f. Decreased peristalsis

a, c, d. The sympathetic nervous system functions under stress to bring about the fight-or-flight response by increasing the heart rate, increasing muscle strength, increasing cardiac output, increasing blood glucose levels, and increasing mental alertness. Increased peristalsis is brought on by the parasympathetic nervous system under normal conditions and at rest.

A nurse is performing patient care for a severely ill patient who has cancer. Which nursing interventions are likely to assist this patient to maintain a positive sense of self? Select all that apply. a. The nurse makes a point to address the patient by name upon entering the room. b. The nurse avoids fatiguing the patient by performing all procedures in silence. c. The nurse performs care in a manner that respects the patient's privacy and sensibilities. d. The nurse offers the patient a simple explanation before moving her in any way. e. The nurse ignores negative feelings from the patient since they are part of the grieving process. f. The nurse avoids conversing with the patient about her life, family, and occupation.

a, c, d. When assisting the patient to maintain a positive sense of self, the nurse should address the patient by name when entering the room; perform care in a manner that respects the patient's privacy; offer a simple explanation before moving the patient's body in any way; acknowledge the patient's status, role, and individuality; and converse with the patient about the patient's life experiences.

A responsibility of the nurse is the administration of preoperative medications to patients. Which statements describe the action of these medications? Select all that apply. a. Diazepam is given to alleviate anxiety. b. Ranitidine is given to facilitate patient sedation. c. Atropine is given to decrease oral secretions. d. Morphine is given to depress respiratory function. e. Cimetidine is given to prevent laryngospasm. f. Fentanyl citrate-droperidol is given to facilitate a sense of calm.

a, c, f. Sedatives, such as diazepam, midazolam, or lorazepam, are given to alleviate anxiety and decrease recall of events related to surgery. Anticholinergics, such as atropine and glycopyrrolate are given to decrease pulmonary and oral secretions and to prevent laryngospasm. Neuroleptanalgesic agents, such as fentanyl citrate-droperidol are given to cause a general state of calm and sleepiness. Histamine-2 receptor blockers, such as cimetidine and ranitidine, are given to decrease gastric acidity and volume. Narcotic analgesics, such as morphine, are given to facilitate patient sedation and relaxation and to decrease the amount of anesthetic agent needed.

A nurse is counseling parents attending a parent workshop on how to build self-esteem in their children. Which teaching points would the nurse include to help parents achieve this goal? Select all that apply. a. Teach the parents to reinforce their child's positive qualities. d. Teach the parents to overlook occasional negative behavior. c. Teach parents to ignore neutral behavior that is a matter of personal preference. d. Teach parents to listen and "fix things" for their children. e. Teach parents to describe the child's behavior and judge it. f. Teach parents to let their children practice skills and make it safe to fail.

a, c, f. The nurse should include the following teaching points for parents: (1) reinforce their child's positive qualities; (2) address negative qualities constructively; (3) ignore neutral behavior that is a matter of taste, preference, or personal style; (4) don't feel they have to "fix things" for their children; (5) describe the child's behavior in a nonjudgmental manner; (6) let their child know what to expect, practice the necessary skills, be patient, and make it safe to fail.

A bile acid sequestrant is the drug of choice for a client who has which conditions? Select all that apply. a. A high LDL concentration b. A high triglyceride concentration c. Biliary obstruction d. Vitamin K deficiency e. A high HDL concentration f. Intolerance to statins

a, f

A nurse practicing in a health care provider's office assesses self-concept in patients during the patient interview. Which patient is least likely to develop problems related to self-concept? a. A 55-year-old television news reporter undergoing a hysterectomy (removal of uterus) b. A young clergyperson whose vocal cords are paralyzed after a motorbike accident c. A 32-year-old accountant who survives a massive heart attack d. A 23-year-old model who just learned that she has breast cancer

a. Based simply on the facts given, the 55-year-old news reporter would be least likely to experience body image or role performance disturbance because she is beyond her childbearing years, and the hysterectomy should not impair her ability to report the news. The young clergyperson's inability to preach, the 32 year old's massive myocardial infarction, and the model's breast resection have much greater potential to result in self-concept problems.

A nurse is counseling a husband and wife who have decided that the wife will get a job so that the husband can go to pharmacy school. Their three teenagers, who were involved in the decision, are also getting jobs to buy their own clothes. The husband, who plans to work 12 to 16 hours weekly, while attending school, states, "I was always an A student, but I may have to settle for Bs now because I don't want to neglect my family." How would the nurse document the husband's self-expectations? a. Realistic and positively motivating his development b. Unrealistic and negatively motivating his development c. Unrealistic but positively motivating his development d. Realistic but negatively motivating his development

a. The patient's self-expectations are realistic, given his multiple commitments, and seem to be positively motivating his development.

A nurse is caring for an older adult in a long-term care facility who has a spinal cord injury affecting his neurologic reflex arc. Based on the patient's condition, what would be a priority intervention for this patient? a. Monitoring food and drink temperatures to prevent burns b. Providing adequate pain relief measures to reduce stress c. Monitoring for depression related to social isolation d. Providing meals high in carbohydrates to promote healing

a. A patient with a damaged neurologic reflex arc would have a diminished pain reflex response, which would put the patient at risk for burns as the sensors in the skin would not detect the heat of the food or liquids. All patients should be provided adequate pain relief, but this is not the priority intervention in this patient. Monitoring for depression would be an intervention for this patient but is not related to the damaged neurologic reflex arc. A patient who is immobile should eat a balanced diet based on the Dietary Guidelines for Americans from the U.S. Department of Health and Human Services and U.S. Department of Agriculture.

A patient had a surgical procedure that necessitated a thoracic incision. The nurse anticipates that the patient will have a higher risk for postoperative complications involving which body system? a. Respiratory system b. Circulatory system c. Digestive system d. Nervous system

a. A thoracic incision makes it more painful for the patient to take deep breaths or cough. Shallow respirations and ineffective coughing increase the risk for respiratory complications.

A nurse teaches problem solving to a college student who is in a crisis situation. What statement best illustrates the student's understanding of the process? a. "I need to identify the problem first." b. "Listing alternatives is the initial step." c. "I will list alternatives after I develop the plan." d. "I do not need to evaluate the outcome of my plan."

a. Although identifying the problem may be difficult, a solution to a crisis situation is impossible until the problem is identified.

A patient who has been in the United States only 3 months has recently suffered the loss of her husband and job. She states that nothing feels familiar—"I don't know who I am supposed to be here"—and says that she "misses home terribly." For what alteration in self-concept is this patient most at risk? a. Personal Identity Disturbance b. Body Image Disturbance c. Self-Esteem Disturbance d. Altered Role Performance

a. An unfamiliar culture, coupled with traumatic life events and loss of husband and job, result in this patient's total loss of her sense of self: "I don't know who I am supposed to be here." Her very sense of identity is at stake, not merely her body image, self-esteem, or role performance.

A 72-year-old woman who is scheduled for a hip replacement is taking several medications on a regular basis. Which drug category might create a surgical risk for this patient? a. Anticoagulants b. Antacids c. Laxatives d. Sedatives

a. Anticoagulant drug therapy would increase the risk for hemorrhage during surgery.

A nurse is providing postmortem care. Which nursing action violates the standards of caring for the body after a patient has been pronounced dead and is not scheduled for an autopsy? a. The nurse leaves the patient in a sitting position while the family visits. b. The nurse places identification tags on both the shroud and the ankle. c. The nurse removes soiled dressings and tubes. d. The nurse makes sure a death certificate is issued and signed.

a. Because the body should be placed in normal anatomic position to avoid pooling of blood, leaving the body in a sitting position is contraindicated. The other actions are appropriate nursing responsibilities related to postmortem care.

A 70-year-old patient who has had a number of strokes refuses further life-sustaining interventions, including artificial nutrition and hydration. She is competent, understands the consequences of her actions, is not depressed, and persists in refusing treatment. Her health care provider is adamant that she cannot be allowed to die this way, and her daughter agrees. An ethics consult has been initiated. Who would be the appropriate decision maker? a. The patient b. The patient's daughter c. The patient's health care provider d. The ethics consult team

a. Because this patient is competent, she has the right to refuse therapy that she finds to be disproportionately burdensome, even if this hastens her death. Neither her daughter nor her doctor has the authority to assume her decision-making responsibilities unless she asks them to do this. The ethics consult team is not a decision-making body; it can make recommendations but has no authority to order anything.

After having an abortion, a patient tells the visiting nurse, "I shouldn't have had that abortion because I'm Catholic, but what else could I do? I'm afraid I'll never get close to my mother or back in the Church again." She then talks with her priest about this feeling of guilt. Which evaluation statement shows a solution to the problem? a. Patient states, "I wish I had talked with the priest sooner. I now know God has forgiven me, and even my mother understands." b. Patient has slept from 10 PM to 6 AM for three consecutive nights without medication. c. Patient has developed mutually caring relationships with two women and one man. d. Patient has identified several spiritual beliefs that give purpose to her life.

a. Because this patient's nursing diagnosis is Spiritual Distress: Guilt, an evaluative statement that demonstrates diminished guilt is necessary. Only answer a directly deals with guilt.

A nurse is caring for a patient with COPD. What would be an expected finding upon assessment of this patient? a. Dyspnea b. Hypotension c. Decreased respiratory rate d. Decreased pulse rate

a. If a problem exists in ventilation, respiration, or perfusion, hypoxia may occur. Hypoxia is a condition in which an inadequate amount of oxygen is available to cells. The most common symptoms of hypoxia are dyspnea (difficulty breathing), an elevated blood pressure with a small pulse pressure, increased respiratory and pulse rates, pallor, and cyanosis.

A nurse caring for patients in a long-term care facility is implementing interventions to help promote sleep in older adults. Which action is recommended for these patients? a. Increase physical activities during the day. b. Encourage short periods of napping during the day. c. Increase fluids during the evening. d. Dispense diuretics during the afternoon hours.

a. In order to promote sleep in the older adult, the nurse should encourage daily physical activity such as walking or water aerobics, discourage napping during the day, decrease fluids at night, and dispense diuretics in the morning or early evening.

A nurse observes involuntary muscle jerking in a sleeping patient. What would be the nurse's next action? a. No action is necessary as this is a normal finding during sleep. b. Call the primary care provider to report possible neurologic deficit. c. Lower the temperature in the patient's room. d. Awaken the patient as this is an indication of night terrors.

a. Involuntary muscle jerking occurs in stage I NREM sleep and is a normal finding. There are no further actions needed for this patient.

To promote sleep in a patient, a nurse suggests what intervention? a. Follow the usual bedtime routine if possible. b. Drink two or three glasses of water at bedtime. c. Have a large snack at bedtime. d. Take a sedative-hypnotic every night at bedtime.

a. Keeping the same bedtime schedule helps promote sleep. Drinking two or three glasses of water at bedtime will probably cause the patient to awaken during the night to void. A large snack may be uncomfortable right before bedtime; instead, a small protein and carbohydrate snack is recommended. Taking a sedative-hypnotic every night disturbs REM and NREM sleep, and sedatives also lose their effectiveness quickly.

A college student visits the school's health center with vague complaints of anxiety and fatigue. The student tells the nurse, "Exams are right around the corner and all I feel like doing is sleeping." The student's vital signs are within normal parameters. What would be an appropriate question to ask in response to the student's verbalizations? a. "Are you worried about failing your exams?" b. "Have you been staying up late studying?" c. "Are you using any recreational drugs?" d. "Do you have trouble managing your time?"

a. Mild anxiety is often handled without conscious thought through the use of coping mechanisms, such as sleeping, which are behaviors used to decrease stress and anxiety. Based on the complaints and normal vital signs, it would be best to explore the patient's level of stress and physiologic response to this stress.

A nurse performing a spiritual assessment collects assessment data from a patient who is homebound and unable to participate in religious activities. Which type of spiritual distress is this patient most likely experiencing? a. Spiritual Alienation b. Spiritual Despair c. Spiritual Anxiety d. Spiritual Pain

a. Spiritual Alienation occurs when there is a "separation from the faith community." Spiritual Despair occurs when the patient is feeling that no one (not even God) cares. Spiritual Anxiety is manifested by a challenged belief and value system, and Spiritual Pain may occur when a patient is unable to accept the death of a loved one.

A hospice nurse is caring for a patient who is terminally ill and who is on a ventilator. After a restless night, the patient hands the nurse a note with the request: "Please help me end my suffering." Which response by a nurse would best reflect adherence to the position of the American Nurses Association (ANA) regarding assisted suicide? a. The nurse promises the patient that he or she will do everything possible to keep the patient comfortable but cannot administer an injection or overdose to cause the patient's death. b. The nurse tells the patient that under no condition can he be removed from the ventilator because this is active euthanasia and is expressly forbidden by the Code for Nurses. c. After exhausting every intervention to keep a dying patient comfortable, the nurse says, "I think you are now at a point where I'm prepared to do what you've been asking me. Let's talk about when and how you want to die." d. The nurse responds: "I'm personally opposed to assisted suicide, but I'll find you a colleague who can help you."

a. The ANA Code of Ethics states that the nurse "should provide interventions to relieve pain and other symptoms in the dying patient consistent with palliative care practice standards and may not act with the sole intent to end life" (2015, p. 3). Yet, nurses may be confronted by patients who seek assistance in ending their lives and must be prepared to respond to the request: "Nurse, please help me die...."

What action does the nurse perform to follow safe technique when using a portable oxygen cylinder? a. Checking the amount of oxygen in the cylinder before using it b. Using a cylinder for a patient transfer that indicates available oxygen is 500 psi c. Placing the oxygen cylinder on the stretcher next to the patient d. Discontinuing oxygen flow by turning the cylinder key counterclockwise until tight

a. The cylinder must always be checked before use to ensure that enough oxygen is available for the patient. It is unsafe to use a cylinder that reads 500 psi or less because not enough oxygen remains for a patient transfer. A cylinder that is not secured properly may result in injury to the patient. Oxygen flow is discontinued by turning the valve clockwise until it is tight.

A patient diagnosed with breast cancer who is in the end stages of her illness has been in the medical intensive care unit for 3 weeks. Her husband tells the nurse that he and his wife often talked about the end of her life and that she was very clear about not wanting aggressive treatment that would merely prolong her dying. The nurse could suggest that the husband speak to his wife's health care provider about which type of order? a. Comfort Measures Only b. Do Not Hospitalize c. Do Not Resuscitate d. Slow Code Only

a. The nurse could suggest that the husband speak to the health care provider about a Comfort Measures Only order. The wife would want all aggressive treatment to be stopped at this point, and all care to be directed to a comfortable, dignified death. A Do Not Hospitalize order is often used for patients in long-term care and other residential settings who have elected not to be hospitalized for further aggressive treatment. A Do Not Resuscitate order means that no attempts are to be made to resuscitate a patient whose breathing or heart stops. A Slow Code means that calling a code and resuscitating the patient are to be delayed until these measures will be ineffectual. Many health care institutions have policies forbidding this, and a nurse could be charged with negligence in the event of a Slow Code and resulting patient death.

A home health care nurse has been visiting a patient with AIDS who says, "I'm no longer afraid of dying. I think I've made my peace with everyone, and I'm actually ready to move on." This reflects the patient's progress to which stage of death and dying? a. Acceptance b. Anger c. Bargaining d. Denial

a. The patient's statement reflects the acceptance stage of death and dying defined by Kübler-Ross.

A nurse is suctioning an oropharyngeal airway for a patient who vomits when it is inserted. Which priority nursing action should be performed by the nurse related to this occurrence? a. Remove the catheter. b. Notify the primary care provider. c. Check that the airway is the appropriate size for the patient. d. Place the patient on his or her back.

a. When a patient vomits upon suctioning of an oropharyngeal airway, the nurse should remove the catheter; it has probably entered the esophagus inadvertently. If the patient needs to be suctioned again, the nurse should change the catheter, because it is probably contaminated. The nurse should also turn the patient to the side and elevate the head of the bed to prevent aspiration.

A client complains about the stress of having to work long hours and missing daily exercise routines. Which response by the nurse is appropriate? a. "Drinking a small glass of wine each day does help reduce stress." b. "Exercise helps reduce the impact of stress on the body and would be a good thing." c. "Maybe exercising, with all of the work, would be too much for your body anyway." d. "There are other ways to reduce stress, such as meditation."

b

A client taking an ACE inhibitor is scheduled for surgery. Because this medication may be dangerous in the setting of general anesthesia, the nurse should a. stop the drug without discussing with the providers. b. alert the provider caring for the patient, and mark the client's chart prominently. c. cancel the surgery and consult with the prescriber. d. monitor fluid levels and make sure the fluids are restricted before surgery.

b

A patient taking a behind-the-counter cold medication and a behind-the-counter allergy medicine is found to be taking double doses of pseudoephedrine. As a result, the patient might exhibit a. ear pain and eye redness. b. restlessness and palpitations. c. sinus pressure and ear pain. d. an irritating cough and nasal drainage.

b

A patient with COPD would be expected to have a. an acute viral infection of the respiratory tract. b. loss of protective respiratory mechanisms due to prolonged irritation or damage. c. localized swelling and inflammation within the lungs. d. inflammation or swelling of the sinus membranes over a prolonged period.

b

A patient with many adverse reactions to drugs is tried on an inhaled steroid for treatment of bronchospasm. For the first 3 days, the patient does not notice any improvement. You should a. switch the patient to a xanthine. b. encourage the patient to continue the drug for 2 to 3 weeks. c. switch the patient to a sympathomimetic. d. try the patient on surfactant.

b

A thrombolytic agent would be most indicated for which circumstance? a. CVA within the last 2 months b. Acute MI within the last 3 hours c. Recent, serious GI bleeding d. Obstetric delivery

b

An older adult client is diagnosed with cardiomyopathy and a cardiac dysrhythmia. What would the nurse expect to be prescribed for this client? a. Digoxin b. Beta blocker c. Fluids d. Nitrate medications

b

Angina a. causes death of heart muscle cells. b. is pain due to lack of oxygen to myocardial cells. c. cannot occur at rest. d. is not treatable.

b

Baroreceptors are the most important factor in controlling fast changes in BP. Baroreceptors a. are evenly distributed throughout the body to maintain pressure in the system. b. sense pressure and immediately send that information to the medulla in the brain. c. are directly connected to the sympathetic nervous system. d. are as sensitive to oxygen levels as to pressure changes.

b

Calcium channel blockers are effective in treating angina because they a. prevent any CV exercise, preventing strain on the heart. b. block strong muscle contractions, causing vasodilation. c. alter the electrolyte balance of the heart, preventing arrhythmias. d. increase the heart rate, making it more efficient.

b

Cardiac cells differ from skeletal muscle cells in that a. they contain actin and myosin. b. they possess automaticity and conductivity. c. calcium must be present for muscle contraction to occur. d. they do not require oxygen to survive.

b

Essential hypertension is the most commonly diagnosed form of high BP. Essential hypertension is a. caused by a tumor in the adrenal gland. b. associated with no known cause. c. related to renal disease. d. caused by liver dysfunction.

b

HMG-CoA reductase inhibitors work in the a. process of bile secretion. b. process of cholesterol formation in the cell. c. intestinal wall to block fat absorption. d. kidney to block fat excretion.

b

Prinzmetal angina occurs as a result of a. electrolyte imbalance. b. a spasm of a coronary vessel. c. decreased venous return to the heart. d. a ventricular arrhythmia.

b

The formation of atheromas in blood vessels precedes the signs and symptoms of a. hepatitis. b. CAD. c. diabetes mellitus. d. chronic obstructive pulmonary disease (COPD).

b

The most dangerous period for many patients undergoing general anesthesia is during which stage? a. Stage 1, when communication becomes difficult b. Stage 2, when systemic stimulation occurs c. Stage 3, when skeletal muscles relax d. There is no real danger during general anesthesia

b

The nurse suspects that a healthy client could be experiencing stress because of which laboratory result? a. Serum potassium of 4.0 mEq/L b. Serum glucose of 165 mg/dL c. Serum calcium of 10.2 mEq/L d. Serum sodium of 142 mEq/L

b

Warfarin, an oral anticoagulant, acts a. to directly prevent the conversion of prothrombin to thrombin. b. to decrease the production of vitamin K clotting factors in the liver. c. as a catalyst in the conversion of plasminogen to plasmin. d. immediately, so it is the drug of choice in emergency situations.

b

Which is true of warfarin? a. Side effects include increased risk of clotting. b. Therapy may take multiple days of dosing to become therapeutic. c. It works by inhibiting activation of factor X. d. It is only administered IV.

b

Which way that the body uses cholesterol stated by a community group indicates that the group has understood teaching about cholesterol? a. The production of water-soluble vitamins b. The formation of steroid hormones c. The mineralization of bones d. The development of dental plaques

b

While a patient is receiving a general anesthetic, he or she must be continually monitored because a. the patient has no pain sensation. b. generalized CNS depression affects all body functions. c. the patient cannot move. d. the patient cannot communicate.

b

You might question an order for a MAOI as a first step in the treatment of depression remembering that these drugs are reserved for use in cases in which there has been no response to other agents because MAOIs a. can cause hair loss. b. are associated with potentially serious drug-food interactions. c. are mostly recommended for use in surgical patients. d. are more expensive than other agents.

b

A nurse is admitting a client to the oncology unit. During the admission assessment, when the nurse asks the client about religious preference, the client states, "I am an atheist." The nurse is aware that the client holds which belief? a. The client believes that there is one God. b. The client does not believe in any god. c. The client believes that there is more than one god. d. The client believes that the existence of God has not been proven.

b An atheist is an individual who does not believe in any god. Monotheism is the belief in the existence of one God. Polytheism is the belief in more than one god. An agnostic is an individual who doubts the existence of God or a supreme being or who believes that the existence of God has not been proven.

A client says that even though a diagnosis of hypertension is disappointing, with medication and lifestyle changes, it can be controlled and the client will become a better person. Based on this data, which aspect of spirituality is the client demonstrating? a. Value b. Becoming c. Connecting d. Meaning

b There are five aspects of spirituality. Becoming involves reflection and allowing life to unfold to know who one is. This is what the client is demonstrating. Value is having cherished beliefs and standards. Meaning is having purpose. Connecting is relating to others or to an Ultimate Other.

While teaching a wellness class on the warning signs of stroke, a participant asks the nurse, "What's the most important thing for me to remember?" What is an appropriate response by the nurse a. "Call 911 if you notice a gradual onset of paralysis or confusion." b. "Be alert for sudden weakness or numbness." c. "Keep a list of your medications." d. "Know your family history."

b Warning signs of stroke include sudden weakness, numbness, paralysis, loss of speech, confusion, dizziness, unsteadiness, and loss of balance-the key word is sudden. Family history and past medical history can be indicators for risk, but they are not warning signs of stroke. Gradual onset of symptoms is not indicative of a stroke.

A client is informed that a surgical procedure is to be scheduled in 2 weeks. Which teaching points should the nurse focus to prepare the client for the surgery? Select all that apply. a. Maintaining a patent airway b. Caring for the surgical incision c. Managing constipation d. Deep breathing and coughing e. Managing pain

b c d e

The postoperative care nurse reviews the documentation from the intraoperative phase and determines that several areas are missing. Which areas did the nurse identify as being missing from the intraoperative documentation? Select all that apply. a. Medication review b. Start and stop times of anesthesia c. Antibiotic infusion times d. Pain assessment e. Start and stop times of the procedure

b c e

The nurse is discussing the various people whom the client will see when in the operating room suite. Which individuals should the nurse emphasize when discussing this aspect of the surgical process with the client? Select all that apply. a. Social worker b. Circulating nurse c. Postoperative nurse d. Anesthesiologist e. Surgeon

b d e

Plasminogen is converted to plasmin, a clot-dissolving substance, by which? Select all that apply. a. Nicotine b. Hageman factor c. Tenecteplase d. Pyrogens e. Thrombin f. Christmas factor

b, c, d

Clients who are using inhalers require careful teaching about which information? Select all that apply. a. Avoiding food 1 hour before and 2 hours after dosing b. Storage of the drug c. Administration techniques to promote therapeutic effects and avoid adverse effects d. Lying flat for as long as 2 hours after dosing e. Timing of administration f. The difference between rescue treatment and prophylaxis

b, c, e, f

A nurse is providing discharge teaching for patients regarding their medications. For which patients would the nurse recommend actions to promote sleep? Select all that apply. a. A patient who is taking iron supplements for anemia. b. A patient with Parkinson's disease who is taking dopamine. c. An older adult taking diuretics for congestive heart failure. d. A patient who is taking antibiotics for an ear infection. e. A patient who is prescribed antidepressants. f. A patient who is taking low-dose aspirin prophylactically.

b, c, e. Drugs that decrease REM sleep include barbiturates, amphetamines, and antidepressants. Diuretics, antiparkinsonian drugs, some antidepressants and antihypertensives, steroids, decongestants, caffeine, and asthma medications are seen as additional common causes of sleep problems.

A nurse working in a long-term care facility is providing teaching to patients with altered oxygenation due to conditions such as asthma and COPD. Which measures would the nurse recommend? Select all that apply. a. Refrain from exercise. b. Reduce anxiety. c. Eat meals 1 to 2 hours prior to breathing treatments. d. Eat a high-protein/high-calorie diet. e. Maintain a high-Fowler's position when possible. f. Drink 2 to 3 pints of clear fluids daily.

b, d, e. When caring for patients with COPD, it is important to create an environment that is likely to reduce anxiety and ensure that they eat a high-protein/high-calorie diet. People with dyspnea and orthopnea are most comfortable in a high-Fowler's position because accessory muscles can easily be used to promote respiration. Patients with COPD should pace physical activities and schedule frequent rest periods to conserve energy. Meals should be eaten 1 to 2 hours after breathing treatments and exercises, and drinking 2 to 3 quarts (1.9 to 2.9 L) of clear fluids daily is recommended.

For gas exchange to occur in the lungs, oxygen must pass through which structures? Select all that apply. a. The conducting airways b. The alveolar epithelium c. The pleural fluid d. The interstitial alveolar wall e. The capillary basement membrane f. The interstitial space

b, e, f

A patient tells a nurse that he would like to appoint his daughter to make decisions for him should he become incapacitated. What should the nurse suggest he prepare? a. POLST form b. Durable power of attorney for health care c. Living will d. Allow Natural Death (AND) form

b. A durable power of attorney for health care appoints an agent the person trusts to make decisions in the event of subsequent incapacity. Living wills provide specific instructions about the kinds of health care that should be provided or foregone in particular situations. A Physician Order for Life-Sustaining Treatment form, or POLST form, is a medical order indicating a patient's wishes regarding treatments commonly used in a medical crisis. The living will is a document whose precise purpose is to allow people to record specific instructions about the type of health care they would like to receive in particular end-of-life situations. Allow natural death on the medical record of a patient indicates the patient or surrogate has expressed a wish that there be no attempts to resuscitate the patient.

A 16-year-old patient has been diagnosed with Body Image Disturbance related to severe acne. In planning nursing care, what is an appropriate goal for this patient? a. The patient will make above-B grades in all tests at school. b. The patient will demonstrate, by diet control and skin care, increased interest in control of acne. c. The patient reports that she feels more self-confident in her music and art, which she enjoys. d. The patient expresses that she is very smart in school.

b. All of these patient goals may be appropriate for the patient, but the only goal that directly addresses her body image disturbance is "the patient will demonstrate by diet control and skin care, increased interest in control of acne."

A nurse is interviewing a patient who just received a diagnosis of pancreatic cancer. The patient tells the nurse "I would never be the type to get cancer; this must be a mistake." Which defense mechanism is this patient demonstrating? a. Projection b. Denial c. Displacement d. Repression

b. Denial occurs when a person refuses to acknowledge the presence of a condition that is disturbing, in this case receiving a diagnosis of pancreatic cancer. Projection involves attributing thoughts or impulses to someone else. Displacement occurs when a person transfers an emotional reaction from one object or person to another object or person. Repression is used by a person to voluntarily exclude an anxiety-producing event from conscious awareness. In the case described in question 9, the patient is not blocking out the fact that the diagnosis was made, the patient is refusing to believe it.

A nurse who is comfortable with spirituality is caring for patients who need spiritual counseling. Which nursing action would be most appropriate for these patients? a. Calling the patient's own spiritual adviser first b. Asking whether the patient has a spiritual adviser the patient wishes to consult c. Attempting to counsel the patient and, if unsuccessful, making a referral to a spiritual adviser d. Advising the patient and spiritual adviser concerning health options and the best choices for the patient

b. Even when a nurse feels comfortable discussing spiritual concerns, the nurse should always check first with patients to determine whether they have a spiritual adviser they would like to consult. Calling the patient's own spiritual adviser may be premature if it is a matter the nurse can handle. The other two options deny patients the right to speak privately with their spiritual adviser from the outset, if this is what they prefer.

A nurse on a maternity ward is teaching new mothers about the sleep patterns of infants and how to keep them safe during this stage. What comment from a parent alerts the nurse that further teaching is required? a. "I can expect my newborn to sleep an average of 16 to 24 hours a day." b. "If I see eye movements or groaning during my baby's sleep I will call the pediatrician." c. "I will place my infant on his back to sleep." d. "I will not place pillows or blankets in the crib to prevent suffocation."

b. Eye movements, groaning, grimacing, and moving are normal activities at this age and would not require a call to the pediatrician. Newborns sleep an average of 16 to 24 hours a day. Infants should be placed on their backs for the first year to prevent SIDS. Parents should be cautioned about placing pillows, crib bumpers, quilts, stuffed animals, and so on in the crib as it may pose a suffocation risk.

A nurse is caring for an obese patient who has had surgery. The nurse monitors this patient for what postoperative complication? a. Anesthetic agent interactions b. Impaired wound healing c. Hemorrhage d. Gas pains

b. Fatty tissue has a poor blood supply and, therefore, has less resistance to infection. As a result, postoperative complications of delayed wound healing, wound infection, and disruption in the integrity of the wound are more common. Patients with a large habitual intake of alcohol require larger doses of anesthetic agents and postoperative analgesics, increasing the risk for drug-related complications. Patients who use illicit drugs are at risk for interactions with anesthetic agents. These are specific to the illicit drug used and should be noted on the medical record for safe anesthetic management. Patients taking anticoagulants are at increased risk for hemorrhage. Gas pains are a common postoperative discomfort.

A patient tells the nurse she is having pain in her right lower leg. How does the nurse determine if the patient has developed a deep vein thrombosis (DVT)? a. By palpating the skin over the tibia and fibula b. By documenting daily calf circumference measurements c. By recording vital signs obtained four times a day d. By noting difficulty with ambulation

b. Manifestations of DVT are pain and cramping in the calf or thigh of the involved extremity, redness and swelling in the affected area, elevated temperature, and an increase in the diameter of the involved extremity. This increase in extremity circumference (typically the calf) is the most significant sign of a DVT and the provider should be notified. The priority for the patient with a known DVT is preventing a clot from breaking loose and becoming a VTE that propagates (travels) to the heart, brain, or lungs called a pulmonary embolism. Thrombophlebitis is an inflammation of a vein associated with thrombus (blood clot) formation. Thrombophlebitis is typically superficial and, in patients without an underlying condition, is often related to IV catheters.

A nurse is performing a sleep assessment on a patient being treated for a sleep disorder. During the assessment, the patient falls asleep in the middle of a conversation. The nurse would suspect which disorder? a. Circadian rhythm sleep-wake disorder b. Narcolepsy c. Enuresis d. Sleep apnea

b. Narcolepsy is an uncontrollable desire to sleep; the person may fall asleep in the middle of a conversation. Circadian rhythm sleep-wake disorders are characterized by a chronic or recurrent pattern of sleep-wake rhythm disruption primarily caused by an alteration in the internal circadian timing system or misalignment between the internal circadian rhythm and the sleep-wake schedule desired or required; a sleep-wake disturbance (e.g., insomnia or excessive sleepiness); and associated distress or impairment, lasting for a period of at least 3 months (except for jet lag disorder) (Sateia, 2014). Enuresis is urinating during sleep or bedwetting. Sleep apnea is a condition in which breathing ceases for a period of time between snoring.

A nurse is teaching a patient a relaxation technique. Which statement demonstrates the need for additional teaching? a. "I must breathe in and out in rhythm." b. "I should take my pulse and expect it to be faster." c. "I can expect my muscles to feel less tense." d. "I will be more relaxed and less aware."

b. No matter what the technique, relaxation involves rhythmic breathing, a slower (not a faster) pulse, reduced muscle tension, and an altered state of consciousness.

A nurse is caring for a patient who has been hospitalized for an acute asthma exacerbation. Which testing method might the nurse use to measure the patient's oxygen saturation? a. Thoracentesis b. Pulse oximetry c. Diffusion capacity d. Maximal respiratory pressure

b. Pulse oximetry is used to obtain baseline information about the patient's oxygen saturation level and is also performed for patients with asthma. Diffusion capacity estimates the patient's ability to absorb alveolar gases and determines if a gas exchange problem exists. Maximal respiratory pressures help evaluate neuromuscular causes of respiratory dysfunction. Both tests are usually performed by a respiratory therapist. The physician or other advanced practice professional can perform a thoracentesis at the bedside with the nurse assisting, or in the radiology department.

Older adults often have reduced vital capacity as a result of normal physiologic changes. Which nursing intervention would be most important for the postoperative care of an older surgical patient specific to this change? a. Take and record vital signs every shift b. Turn, cough, and deep breathe every 4 hours c. Encourage increased intake of oral fluids d. Assess bowel sounds daily

b. Reduced vital capacity in older adults increases the risk for respiratory complications, including pneumonia and atelectasis. Having the patient turn, cough, and deep breathe every 4 hours maintains respiratory function and helps to prevent complications.

A nurse caring for patients in a busy hospital environment should implement which recommendation to promote sleep? a. Keep the room light dimmed during the day. b. Keep the room cool. c. Keep the door of the room open. d. Offer a sleep aid medication to patients on a regular basis.

b. The nurse should keep the room cool and provide earplugs and eye masks. The nurse should also maintain a brighter room environment during daylight hours and dim lights in the evening, and keep the door of the room closed. Sleep aid medications should only be offered as prescribed.

A nurse is caring for terminally ill patients in a hospital setting. Which nursing action describes appropriate end-of-life care? a. To eliminate confusion, the nurse takes care not to speak too much when caring for a comatose patient. b. The nurse sits on the side of the bed of a dying patient, holding the patient's hand, and crying with the patient. c. The nurse refers to a counselor the daughter of a dying patient who is complaining about the care associated with artificially feeding her father. d. The nurse tells a dying patient to sit back and relax and performs patient hygiene for the patient because it is easier than having the patient help.

b. The nurse should not be afraid to show compassion and empathy for the dying person, including crying with the patient if it occurs. The sense of hearing is believed to be the last sense to leave the body, and many patients retain a sense of hearing almost to the moment of death; therefore, nurses should explain to the comatose patient the nursing care being given. The nurse should address caregiver role endurance by actively listening to family members. Because it is good to encourage dying patients to be as active as possible for as long as possible, it is generally not good practice to perform basic self-care measures the patient can perform simply because it is "easier" to do it this way.

A nurse is choosing a catheter to use to suction a patient's endotracheal tube via an open system. On which variable would the nurse base the size of the chosen catheter? a. The age of the patient b. The size of the endotracheal tube c. The type of secretions to be suctioned d. The height and weight of the patient

b. The nurse would base the size of the suctioning catheter on the size of the endotracheal tube. The external diameter of the suction catheter should not exceed half of the internal diameter of the endotracheal tube. Larger catheters can contribute to trauma and hypoxemia.

A nurse asks a 25-year-old patient to describe himself with a list of 20 words. After 15 minutes, the patient listed "25 years old, male, named Joe," then declared he couldn't think of anything else. What should the nurse document regarding this patient? a. Lack of self-esteem b. Deficient self-knowledge c. Unrealistic self-expectation d. Inability to evaluate himself

b. The patient's inability to list more than three items about himself indicates deficient self-knowledge. There are not enough data provided to determine whether he lacks self-esteem, has unrealistic self-expectations, or is unable to evaluate himself.

A 70-year-old male is scheduled for surgery. He says to the nurse, "I am so frightened—what if I don't wake up?" What would be the nurse's best response? a. "You have a wonderful doctor." b. "Let's talk about how you are feeling." c. "Everyone wakes up from surgery!" d. "Don't worry, you will be just fine."

b. This answer allows the patient to talk about feelings and fears, and is therapeutic.

A college freshman away from home for the first time says to a counselor, "Why did I have to be born into a family of big bottoms and short fat legs! No one will ever ask me out for a date. Oh, why can't I have long thin legs like everyone else in my class? What a frump I am." What type of disturbance in self-concept is this patient experiencing? a. Personal Identity Disturbance b. Body Image Disturbance c. Self-Esteem Disturbance d. Altered Role Performance

b. This patient's concern is with body image. The information provided does not suggest a nursing diagnosis of Personal Identity Disturbance, Self-Esteem Disturbance, or Altered Role Performance.

A nurse is visiting a patient with pancreatic cancer who is dying at home. During the visit, he breaks down and cries, and tells the nurse that it is unfair that he should have to die now when he's finally made peace with his family. Which response by the nurse would be most appropriate? a. "You can't be feeling this way. You know you are going to die." b. "It does seem unfair. Tell me more about how you are feeling." c. "You'll be all right; who knows how much time any of us has." d. "Tell me about your pain. Did it keep you awake last night?"

b. This response by the nurse validates that what the patient is saying has been heard and invites him to share more of his feelings, concerns, and fears. The other responses either deny the patient's feelings or change the subject.

A nurse is caring for a terminally ill patient during the 11 PM to 7 AM shift. The patient says, "I just can't sleep. I keep thinking about what my family will do when I am gone." What response by the nurse would be most appropriate? a. "Oh, don't worry about that now. You need to sleep." b. "What seems to be concerning you the most?" c. "I have talked to your wife and she told me she will be fine." d. "I'm not qualified to advise you, I suggest you discuss this with your wife."

b. Using an open-ended question allows the patient to continue talking. An open-ended question, such as, "What seems to be concerning you the most?" provides a means of encouraging communication. False reassurances are not helpful. Also, the patient's feelings and restlessness should be addressed as soon as possible.

An emergency department nurse is using a manual resuscitation bag (Ambu bag) to assist ventilation in a patient with lung cancer who has stopped breathing on his own. What is an appropriate step in this procedure? a. Tilt the patient's head forward. b. Hold the mask tightly over the patient's nose and mouth. c. Pull the patient's jaw backward. d. Compress the bag twice the normal respiratory rate for the patient.

b. With the patient's head tilted back, jaw pulled forward, and airway cleared, the mask is held tightly over the patient's nose and mouth. The bag also fits easily over tracheostomy and endotracheal tubes. The operator's other hand compresses the bag at a rate that approximates normal respiratory rate (e.g., 16 to 20 breaths/min in adults).

A client is brought to the emergency room with a suspected myocardial infarction. The client is upset because he had just had an ECG in his doctor's office and it was fine. The explanation of this common phenomenon would include the fact that a. the ECG only reflects changes in cardiac output. b. the ECG is not a very accurate test. c. the ECG only measures the flow of electrical current through the heart. d. the ECG is not related to the heart problems.

c

A college student tells the nurse about being "out of control" with eating. The client states, "I am trying to keep my weight down so my mom does not call me fat. I make myself throw up after eating." Based on this data, which disorder does the nurse use when planning care for this client? a. Binge-eating b. Anorexia nervosa c. Bulimia d. Purging disorder

c

A heart transplantation patient has no nerve connections to the transplanted heart. In such an individual, one would expect to find a. a slower-than-normal resting heart rate. b. atria that contract at a different rate than ventricles. c. an increase in heart rate during emotional stress. d. inability to exercise because there is no way to increase heart rate.

c

A patient is not getting a response to the antihistamine that was prescribed. Appropriate action might include a. switching to a decongestant. b. stopping the drug and increasing fluids. c. trying a different antihistamine. d. switching to a corticosteroid.

c

A patient who is hypertensive becomes pregnant. Which is the safest medication for this patient? a. An angiotensin II receptor blocker b. An ACE inhibitor c. A beta-blocker d. A calcium-channel blocker

c

A pediatric patient is prescribed phenobarbital preoperatively to relieve anxiety and produce sedation. After giving the injection, you should assess the patient for a. acute Stevens-Johnson syndrome. b. bone marrow depression. c. paradoxical excitement. d. withdrawal syndrome.

c

A person who could benefit from an anxiolytic drug for short-term treatment of insomnia would not be prescribed a. zolpidem. b. zaleplon. c. buspirone. d. meprobamate.

c

ACE inhibitors work on the renin-angiotensin system to prevent the conversion of angiotensin I to angiotensin II. Because this blocking occurs in the cells in the lung, which is usually the site of this conversion, the use of ACE inhibitors often results in a. spontaneous pneumothorax. b. pneumonia. c. unrelenting cough. d. respiratory depression.

c

An older adult client diagnosed with asthma has a respiratory rate of 28 at rest with audible wheezes upon inspiration. Based on this data, which nursing diagnosis is the most appropriate? a. Ineffective Airway Clearance b. Activity Intolerance c. Ineffective Breathing Pattern d. Impaired Tissue Perfusion

c

Antitussives are useful in blocking the cough reflex and preserving the energy associated with prolonged, nonproductive coughing. Antitussives are best used with a. postoperative patients. b. asthma patients. c. patients with a dry, irritating cough. d. COPD patients who tire easily.

c

Anxiolytic drugs block the awareness of and reaction to the environment. This effect would not be beneficial a. to relieve extreme fear. b. to moderate anxiety related to unknown causes. c. in treating a patient who must drive a vehicle for a living. d. in treating a patient who is experiencing a stress reaction.

c

Blood coagulation is a complex reaction that involves a. vasoconstriction, platelet aggregation, and plasminogen action. b. vasodilation, platelet aggregation, and activation of the clotting cascade. c. vasoconstriction, platelet aggregation, and conversion of prothrombin to thrombin. d. vasodilation, platelet inhibition, and action of the intrinsic and extrinsic clotting cascades.

c

Blood flow to the myocardium differs from blood flow to the rest of the cells of the body in that a. blood perfuses the myocardium during systole. b. blood flow is determined by many local factors, including buildup of acid. c. blood perfuses the myocardium during diastole. d. oxygenated blood flows to the myocardium via veins.

c

Depression is an affective disorder that is a. always precipitated by a specific event. b. most common in patients with head injuries. c. characterized by overwhelming sadness, despair, and hopelessness. d. very evident and easy to diagnose in the clinical setting.

c

Heparin reacts to prevent the conversion of prothrombin to thrombin. Heparin a. is available in oral and parenteral forms. b. takes about 72 hours to have a therapeutic effect. c. has its effects reversed with the administration of protamine sulfate. d. has its effects reversed with the injection of vitamin K.

c

Hyperlipidemia is considered to be a. a normal finding in adult males. b. related to stress levels. c. a treatable CAD risk factor. d. a side effect of cigarette smoking.

c

In the heart, a. the ventricles will not contract unless they are stimulated by action potentials arising from the SA node. b. fibrillation of the atria will cause blood pressure to fall to zero. c. spontaneous depolarization of the muscle membrane can occur in the absence of nerve stimulation. d. the muscle can continue to contract for a long period of time in the absence of oxygen.

c

Recovery after a general anesthetic refers to the period of time a. from the beginning of anesthesia until the patient is ready for surgery. b. during the surgery when anesthesia is maintained at a certain level. c. from discontinuation of the anesthetic until the patient has regained consciousness, movement, and the ability to communicate. d. when the patient is in the most danger of CNS depression.

c

Respiratory distress syndrome occurs in a. babies with frequent colds. b. babies with genetic allergies. c. premature and low-birth-weight babies. d. babies stressed during the pregnancy.

c

The biogenic amine theory of depression states that depression is a result of a. an unpleasant childhood. b. gamma-aminobutyric acid (GABA) inhibition. c. deficiency of NE, dopamine, or 5HT in key areas of the brain. d. blockages within the limbic system, which controls emotions and affect.

c

The nurse emphasizes the need to take sinusitis seriously because a. it can cause a loss of sleep and exhaustion. b. it can lead to a painful otitis media. c. if it is left untreated, microorganisms can travel to brain tissue. d. drainage from infected sinus membranes often leads to pneumonia.

c

The nurse is planning care for a client experiencing dyspnea and a subsequent activity intolerance. Which action by the nurse is the most appropriate? a. Encourage strenuous activity. b. Consult a dietitian for low-calorie meals. c. Consult physical therapy for endurance and musculoskeletal function d. Encourage dependence with activities of daily living.

c

The nurse is providing care to a client who has a tracheostomy. The nurse will monitor the client for complications related to the loss of which protective mechanism? a. The ability to cough b. Decrease in oxygen-carrying capacity of the trachea c. Filtration and humidification of inspired air d. The sneeze reflex initiated by irritants in the nasal passages

c

The nursing instructor determines that teaching about general anesthetics was successful when the students identify which person as being most qualified to administer general anesthetics? a. Nursing supervisor b. Graduate nurse c. Trained physician d. Surgeon

c

The type II cells of the walls of the alveoli function to a. replace mucus in the alveoli. b. produce serotonin. c. secrete surfactant. d. protect the lungs from bacterial invasion.

c

The xanthines a. block the sympathetic nervous system. b. stimulate the sympathetic nervous system. c. directly affect the smooth muscles of the respiratory tract. d. act in the CNS to cause bronchodilation.

c

When auscultating the lungs of a client experiences dyspnea, the nurse hears a low-pitched sound that is continuous throughout inspiration. What does this lung sound indicate to the nurse? a. Narrow bronchi b. Narrow trachea passages c. Blocked large airway passages d. Inflamed pleural surfaces

c

When describing heart valves to a group of students, would the instructor include? a. The closing of the AV valves is what is solely responsible for heart sounds. b. Small muscles attached to the AV valves are responsible for opening and closing the valves. c. The aortic valve opens when the pressure in the left ventricle becomes greater than the aortic pressure. d. The valves leading to the great vessels are called the cuspid valves.

c

When teaching a patient receiving TCAs, it is important to remember that TCAs are associated with many anticholinergic adverse effects. Teaching about these drugs should include anticipation of a. increased libido and increased appetite. b. polyuria and polydipsia. c. urinary retention, arrhythmias, and constipation. d. hearing changes, cataracts, and nightmares.

c

Which is most critical for respiration to occur? a. Low levels of oxygen b. Low levels of CO2 c. Functioning inspiratory muscles d. An actively functioning autonomic system

c

Which would the nurse expect the healthcare provider to prescribe for a patient who has high lipid levels and cannot take fibrates or HMG-CoA reductase inhibitors? a. Nicotine b. Vitamin C c. PCSK9 inhibitor d. Nitrates

c

Which would the nurse include when teaching a patient about HMG-CoA reductase inhibitors? a. The patient will not have a heart attack. b. The patient will not develop CAD. c. The patient might develop cataracts as a result. d. The patient might stop absorbing fat-soluble vitamins.

c

Your patient has been maintained on theophylline for many years and has recently taken up smoking. The theophylline levels in this patient would be expected to a. rise because nicotine prevents the breakdown of theophylline. b. stay the same because smoking has no effect on theophylline. c. fall because substances in cigarettes stimulate liver metabolism of theophylline. d. rapidly reach toxic levels.

c

An older adult client diagnosed with cardiomyopathy reports having to rest between activities during the day. The client asks the nurse why this is occurring. Which reason should the nurse include in the response to the client? a. Increased stroke volume b. Increased blood pressure c. Decreased cardiac output d. An elongated and dilated aorta

c Decreased cardiac output is a result of decreased efficiency and contractibility of the myocardium. Rest could be required after each activity that puts physiological stress on the heart. Less blood is pumped from the heart to the rest of the body with a decreased cardiac output, and this has a direct effect on the activity level that can be tolerated. It is unknown if the client has high blood pressure, an elongated and dilated aorta, or increased stroke volume.

While hospitalized, a client learns that a dear friend has died as a result of an accident. The client is crying and asking God, "Why?" The nurse realizes the client is demonstrating which type of spiritual distress? a. Physiological b. Treatment-related c. Situational d. Psychological

c Factors that may be associated with or contribute to an individual's spiritual distress include situational concerns, physiologic problems, and treatment-related concerns. Situational factors include the death or illness of a significant other, inability to practice one's spiritual rituals, or feelings of embarrassment when practicing them. Physiologic problems include having a medical diagnosis of a terminal or debilitating disease. Treatment-related factors include recommendation for treatment, surgery, dietary restrictions, or isolation. Psychological is not a factor that contributes to spiritual distress.

The nurse identifies the nursing diagnosis of grieving as appropriate for the family of a terminally ill client. Which family behavior supports this diagnosis? a. The family members are crying out loud and wringing their hands during visits. b. Some family members state they cannot go on with life. c. The family is tearful and sad during visits with the client. d. The family members state that they cannot care for the client at home.

c Grieving prior to the actual loss is termed anticipatory grieving, which the family is demonstrating by being tearful and sad. Loud crying and wringing of hands might be the beginning of complicated grieving because the client is still alive. When the family members state that they cannot go on with life, they are demonstrating hopelessness. Being unable to care for the client in the home is an example of caregiver role strain.

A nurse is preparing a patient for a cesarean section and teaches her the effects of the regional anesthesia she will be receiving. Which effects would the nurse expect? Select all that apply. a. Loss of consciousness b. Relaxation of skeletal muscles c. Reduction or loss of reflex action d. Localized loss of sensation e. Prolonged pain relief after other anesthesia wears off f. Infiltrates the underlying tissues in an operative area

c, d. A localized loss of sensation and possible loss of reflexes occur with a regional anesthetic. Loss of consciousness and relaxation of skeletal muscles occur with general anesthesia. Prolonged pain relief after other anesthesia wears off and infiltration of the underlying tissues in an operative area occur with topical anesthesia.

A client who has nitroglycerin to avert an acute anginal attack would need to be taught... Select all that apply. a. to take five or six tablets and then seek medical help if no relief occurs. b. to buy the tablets in bulk to decrease the cost. c. to protect tablets from light and humidity. d. to store the tablets in a clearly marked, clear container in open view. e. to use the nitroglycerin before an event or activity that will most likely precipitate an anginal attack. f. to discard them if they do not fizzle when placed under the tongue.

c, e, f

A nurse observes a slight increase in a patient's vital signs while he is sleeping during the night. According to the patient's stage of sleep, the nurse expects what conditions to be true? Select all that apply. a. He is aware of his surroundings at this point. b. He is in delta sleep at this time. c. It would be most difficult to awaken him at this time. d. This is most likely an NREM stage. e. This stage constitutes around 20% to 25% of total sleep. f. The muscles are relaxed in this stage.

c, e. This scenario describes REM sleep. During REM sleep, it is difficult to arouse a person, and the vital signs increase. REM sleep constitutes about 20% to 25% of sleep. In stage I NREM sleep, the person is somewhat aware of surroundings. Delta sleep is NREM stages III and IV sleep. In stage IV NREM sleep, the muscles are relaxed, whereas small muscle twitching may occur in REM sleep.

A nurse caring for patients in a hospital setting uses anticipatory guidance to prepare them for painful procedures. Which nursing intervention is an example of this type of stress management? a. The nurse teaches a patient rhythmic breathing to perform prior to the procedure. b. The nurse tells a patient to focus on a pleasant place, mentally place himself in it, and breathe slowly in and out. c. The nurse teaches a patient about the pain involved in the procedure and describes methods to cope with it. d. The nurse teaches a patient to create and focus on a mental image during the procedure in order to be less responsive to the pain.

c. Anticipatory guidance focuses on psychologically preparing a person for an unfamiliar or painful event. When the patient know what to expect—for example, when the nurse tells the patient about the pain he or she should expect to experience during a procedure, and describes related pain relief measures—the patient's anxiety is reduced. Rhythmic breathing is a relaxation technique, focusing on a pleasant place and breathing slowly in and out is a meditation technique, and focusing on a mental image to reduce responses to stimuli is a guided imagery technique.

A nurse is assisting a respiratory therapist with chest physiotherapy for patients with ineffective cough. For which patient might this therapy be recommended? a. A postoperative adult b. An adult with COPD c. A teenager with cystic fibrosis d. A child with pneumonia

c. Chest physiotherapy may help loosen and mobilize secretions, increasing mucus clearance. This is especially helpful for patients with large amounts of secretions or an ineffective cough, such as patients with cystic fibrosis. Chest physiotherapy has limited evidence for its effectiveness and is not recommended for use in numerous patient populations, including children with pneumonia, adults with COPD, and postoperative adults

A nurse who is caring for patients on a pediatric ward is assessing the children for their spiritual needs. Which is the most important source of learning for a child's own spirituality? a. The child's church or religious organization b. What parents say about God and religion c. How parents behave in relationship to one another, their children, others, and to God d. The spiritual adviser for the family

c. Children learn most about their own spirituality from how their parents behave in relationship to one another, their children, others, and God (or a higher being). What parents say about God and religion, the family's spiritual advisor, and the child's church or religious organization are less important sources of learning.

A nurse is caring for a patient who states he has had trouble sleeping ever since his job at a factory changed from the day shift to the night shift. For what recommended treatment might the nurse prepare this patient? a. The use of a central nervous system stimulant b. Continuous positive airway pressure machine (CPAP) c. Chronotherapy d. The application of heat or cold therapy to promote sleep

c. Chronotherapy requires a commitment on the part of the patient to act over a period of weeks to progressively advance or delay the time of sleep for 1 to 2 hours per day. Over time, this results in a shift of the sleep-wake cycle. The use of a central nervous system stimulant is recommended for narcolepsy. Continuous positive airway pressure machine (CPAP) is used for OSA, and the application of heat or cold therapy to the legs is used to treat RLS.

A nurse interviews an 82-year-old resident of a long-term care facility who says that she has never gotten over the death of her son 20 years ago. She reports that her life fell apart after that and she never again felt like herself or was able to enjoy life. What type of grief is this woman experiencing? a. Somatic grief b. Anticipatory grief c. Unresolved grief d. Inhibited grief

c. Dysfunctional grief is abnormal or distorted; it may be either unresolved or inhibited. In unresolved grief, a person may have trouble expressing feelings of loss or may deny them; unresolved grief also describes a state of bereavement that extends over a lengthy period. With inhibited grief, a person suppresses feelings of grief and may instead manifest somatic (body) symptoms, such as abdominal pain or heart palpitations. Somatic grief is not a classification of grief, rather somatic symptoms are the expression of grief that may occur with inhibited grief. Anticipatory loss or grief occurs when a person displays loss and grief behaviors for a loss that has yet to take place.

A school nurse is teaching parents how to foster a healthy development of self in their children. Which statement made by one of the parents needs to be followed up with further teaching? a. "I love my child so much I 'hug him to death' every day." b. "I think children need challenges, don't you?" c. "My husband and I both grew up in very restrictive families. We want our children to be free to do whatever they want." d. "My husband and I have different ideas about discipline, but we're talking this out because we know it's important for Johnny that we be consistent."

c. Each option with the exception of c correctly addresses some aspect of fostering healthy development in children. Because children need effective structure and development, giving them total freedom to do as they please may actually hinder their development.

Mr. Brown's teenage daughter had been involved in shoplifting. He expresses much anger toward her and states he cannot face her, let alone discuss this with her: "I just will not tolerate a thief." Which nursing intervention would the nurse take to assist Mr. Brown with his deficit in forgiveness? a. Assure Mr. Brown that many parents feel the same way. b. Reassure Mr. Brown that many teenagers go through this kind of rebellion and that it will pass. c. Assist Mr. Brown to identify how unforgiving feelings toward others hurt the person who cannot forgive. d. Ask Mr. Brown if he is sure he has spent sufficient time with his daughter.

c. Helping Mr. Brown identify how his unforgiving feelings may be harmful to him is the only nursing intervention that directly addresses his unmet spiritual need concerning forgiveness. Assuring Mr. Brown that many parents would feel the same way or that many teenagers shoplift out of rebelliousness may make him feel better initially, but neither option addresses his need to forgive. Suggesting that Mr. Brown may not have spent enough time with his daughter is likely to make him feel guilty.

A nurse working the night shift in a pediatric unit observes a 10-year-old patient who is snoring and appears to have labored breathing during sleep. Upon reporting the findings to the primary care provider, what nursing action might the nurse expect to perform? a. Preparing the family for a diagnosis of insomnia and related treatments. b. Preparing the family for a diagnosis of narcolepsy and related treatments. c. Anticipating the scheduling of polysomnography to confirm OSA. d. No action would be taken, as this is a normal finding for hospitalized children.

c. OSA (pediatric) is defined by the presence of one of these findings: snoring, labored/obstructed breathing, enuresis, or daytime consequences (hyperactivity or other neurobehavioral problems, sleepiness, fatigue). According to the American Academy of Pediatrics children and adolescents with symptoms of OSA, including snoring, should have polysomnography to confirm the diagnosis. Although OSA may cause insomnia, this is not the primary diagnosis in this case. Narcolepsy is a condition characterized by excessive daytime sleepiness and frequent overwhelming urges to sleep or inadvertent daytime lapses into sleep. This scenario is not usually a normal finding in hospitalized children during sleep.

A nurse working in an emergency department assesses how patients' religious beliefs affect their treatment plan. With which patient would the nurse be most likely to encounter resistance to emergency lifesaving surgery? a. A patient of the Adventist faith b. A patient who practices Buddhism c. A patient who is a Jehovah's Witness d. A patient who is an Orthodox Jew

c. Patients who practice the Jehovah's Witness faith believe blood transfusions violate God's laws and do not allow them. The other religious groups do not restrict modern lifesaving treatment for their members.

A nurse is performing spirituality assessments of patients living in a long-term care facility. What is the best question the nurse might use to assess for spiritual needs? a. Can you describe your usual spiritual practices and how you maintain them daily? b. Are your spiritual beliefs causing you any concern? c. How can I and the other nurses help you maintain your spiritual practices? d. How do your religious beliefs help you to feel at peace?

c. Questioning how the staff can meet patients' spiritual practices assesses spiritual needs. Asking the patient to describe spiritual practices assesses spiritual practices. Asking about concerns assesses spiritual distress, and asking about feeling at peace assesses the need for forgiveness.

A nurse is assessing the developmental levels of patients in a pediatric office. Which person would a nurse document as experiencing developmental stress? a. An infant who learns to turn over b. A school-aged child who learns how to add and subtract c. An adolescent who is a "loner" d. A young adult who has a variety of friends

c. The adolescent who is a loner is not meeting a major task (being a part of a peer group) for that level of growth and development.

A nurse is caring for a patient in the shock or alarm reaction phase of the GAS. Which response by the patient would be expected? a. Decreasing pulse b. Increasing sleepiness c. Increasing energy levels d. Decreasing respirations

c. The body perceives a threat and prepares to respond by increasing the activity of the autonomic nervous and endocrine systems. The initial or shock phase is characterized by increased energy levels, oxygen intake, cardiac output, blood pressure, and mental alertness.

A nurse is performing a psychological assessment of a 19-year-old patient who has Down's syndrome. The patient is mildly developmentally disabled with an intelligence quotient of 82. He told his nurse, "I'm a good helper. You see I can carry these trays because I'm so strong. But I'm not very smart, so I have just learned to help with the things I know how to do." What findings for self-concept and self-esteem would the nurse document for this patient? a. Negative self-concept and low self-esteem b. Negative self-concept and high self-esteem c. Positive self-concept and fairly high self-esteem d. Positive self-concept and low self-esteem

c. The data point to the patient having a positive self-concept ("I'm a good helper") and fairly high self-esteem (realizes his strengths and limitations). The statement "But I'm not very smart" is accurate and is not an indication of a negative self-concept.

A premature infant with serious respiratory problems has been in the neonatal intensive care unit for the last 3 months. The infant's parents also have a 22-month-old son at home. The nurse's assessment data for the parents include chronic fatigue and decreased energy, guilt about neglecting the son at home, shortness of temper with one another, and apprehension about their continued ability to go on this way. What human response would be appropriate for the nurse to document? a. Grieving b. Ineffective Coping c. Caregiver Role Strain d. Powerlessness

c. The defining characteristics for the NANDA diagnosis Caregiver Role Strain fit the set of assessment data provided. The other diagnoses do not fit the assessment data.

A mother of a 10-year-old daughter tells the nurse: "I feel incompetent as a parent and don't know how to discipline my daughter." What should be the nurse's first intervention when counseling this patient? a. Recommend that she discipline her daughter more strictly and consistently. b. Make a list of things her husband can do to give her more time and help her improve her parenting skills. c. Assist the mother to identify both what she believes is preventing her success and what she can do to improve. d. Explore with the mother what the daughter can do to improve her behavior and make the mother's role as a parent easier.

c. The first intervention priority with a mother who feels incompetent to parent a daughter is to assist the mother to identify what is preventing her from being an effective parent and then to explore solutions aimed at improving her parenting skills. The other interventions may prove helpful, but they do not directly address the mother's problem with her feelings of incompetence.

A nurse is performing an assessment of a woman who is 8 months pregnant. The woman states, "I worry all the time about being able to handle becoming a mother." Which nursing diagnosis would be most appropriate for this patient? a. Ineffective Coping related to the new parenting role b. Ineffective Denial related to ability to care for a newborn c. Anxiety related to change in role status d. Situational Low Self-Esteem related to fear of parenting

c. The most appropriate nursing diagnosis is Anxiety, which indicates situational/maturational crises or changes in role status. Ineffective Coping refers to an inability to appraise stressors or use available resources. Ineffective Denial is a conscious or unconscious attempt to disavow the knowledge or meaning of an event to reduce anxiety, and leads to detriment of health. Situational Low Self-Esteem refers to feelings of worthlessness related to the situation the person is currently experiencing, not to the fear of role changes.

A nurse is caring for a patient with chronic lung disease who is receiving oxygen through a nasal cannula. What nursing action is performed correctly? a. The nurse assures that the oxygen is flowing into the prongs. b. The nurse adjusts the fit of the cannula so it fits snug and tight against the skin. c. The nurse encourages the patient to breathe through the nose with the mouth closed. d. The nurse adjusts the flow rate to 6 L/min or more.

c. The nurse should encourage the patient to breathe through the nose with the mouth closed. The nurse should assure that the oxygen is flowing out of the prongs prior to inserting them into the patient's nostrils. The nurse should adjust the fit of the cannula so it is snug but not tight against the skin. The nurse should adjust the flow rate as ordered.

A nurse is discussing with an older adult patient measures to take to induce sleep. What teaching point might the nurse include? a. Drinking a cup of regular tea at night induces sleep. b. Using alcohol moderately promotes a deep sleep. c. Having a small bedtime snack high in tryptophan and carbohydrates improves sleep. d. Exercising right before bedtime can hinder sleep.

c. The nurse would teach the patient that having a small bedtime snack high in tryptophan and carbohydrates improves sleep. Regular tea contains caffeine and increases alertness. Large quantities of alcohol limit REM and delta sleep. Physical activity within a 3-hour interval before normal bedtime can hinder sleep.

A man who is a declared agnostic is extremely depressed after losing his home, his wife, and his children in a fire. His nursing diagnosis is Spiritual Distress: Spiritual Pain related to inability to find meaning and purpose in his current condition. What is the most important nursing intervention to plan? a. Ask the patient which spiritual adviser he would like you to call. b. Recommend that the patient read spiritual biographies or religious books. c. Explore with the patient what, in addition to his family, has given his life meaning and purpose in the past. d. Introduce the belief that God is a loving and personal God.

c. The nursing intervention of exploring with the patient what, in addition to his family, has given his life meaning and purpose in the past is more likely to correct the etiology of his problem, Spiritual Pain, than any of the other nursing interventions listed.

A nurse is securing a patient's endotracheal tube with tape and observes that the tube depth changed during the retaping. Which action would be appropriate related to this incident? a. Instruct the assistant to notify the primary care provider. b. Assess the patient's vital signs. c. Remove the tape, adjust the depth to ordered depth and reapply the tape. d. No action is required as depth will adjust automatically.

c. The tube depth should be maintained at the same level unless otherwise ordered by the health care provider. If the depth changes, the nurse should remove the tape, adjust the tube to ordered depth, and reapply the tape.

A 33-year-old businessperson is in counseling, attempting to deal with a long-repressed history of sexual abuse by her father. "I guess I should feel satisfied with what I've achieved in life, but I'm never content, and nothing I achieve makes me feel good about myself.... I hate my father for making me feel like I'm no good. This is an awful way to live." What self-concept disturbance is this person experiencing? a. Personal Identity Disturbance b. Body Image Disturbance c. Self-Esteem Disturbance d. Altered Role Performance

c. This patient's self-concept disturbance is mainly one of devaluing herself and thinking that she is no good. This is a Self-Esteem Disturbance.

A perioperative nurse is preparing a patient for surgery for treatment of a ruptured spleen as the result of an automobile crash. For what type of surgery would the nurse prepare this patient? a. Minor, diagnostic b. Minor, elective c. Major, emergency d. Major, palliative

c. This surgery would involve a major body organ, has the potential for postoperative complications, requires hospitalization, and must be done immediately to save the patient's life. Elective surgery is a procedure that is preplanned by essentially healthy people. Diagnostic surgery is performed to confirm a diagnosis. Palliative surgery is not curative, rather it is done to relieve or reduce the intensity of an illness.

A hospice nurse is caring for a patient who is dying of pancreatic cancer. The patient tells the nurse "I feel no connection to God" and "I'm worried that I find no real meaning in life." What would be the nurse's best response to this patient? a. Give the patient a hug and tell him that his life still has meaning. b. Arrange for a spiritual adviser to visit the patient. c. Ask if the patient would like to talk about his feelings. d. Call in a close friend or relative to talk to the patient.

c. When caring for a patient who is in spiritual distress, the nurse should listen to the patient first and then ask whether the patient would like to visit with a spiritual adviser. To arrange for a spiritual adviser first may not respect the wishes of the patient. A hug and false reassurances do not address the diagnosis of spiritual distress. Talking to friends or relatives may be helpful, but only if the patient desires their visits.

A nurse who was raised as a strict Roman Catholic but who is no longer a practicing Catholic stated she couldn't assist patients with their spiritual distress because she recognizes only a "field power" in each person. She said, "My parents and I hardly talk because I've deserted my faith. Sometimes I feel real isolated from them and also from God—if there is a God." Analysis of these data reveals which unmet spiritual need? a. Need for meaning and purpose b. Need for forgiveness c. Need for love and relatedness d. Need for strength for everyday living

c. The data point to an unmet spiritual need to experience love and belonging, given the nurse's estrangement from her family and God after leaving the church. The other options may represent other needs this nurse has, but the data provided do not support them.

A client reports feeling tired and not refreshed after sleeping even when everyone at home laughs about her loud snoring at night. What should the nurse suspect as being the cause of this client's fatigue? a. Insomnia b. Depression c. Thyroid disorder d. Sleep apnea

d

A nurse would question an order for which drug if the patient was also receiving verapamil? a. Oral contraceptives b. Cyclosporine c. Digoxin d. Barbiturate anesthetics

d

A patient who coughs is experiencing a reflex caused by a. inflammation irritating the sinuses in the skull. b. irritants affecting receptor sites in the nasal cavity. c. pressure against the eustachian tube. d. irritation to receptors in the trachea and conducting airways.

d

A patient with sinus pressure and pain related to seasonal rhinitis would benefit from taking a. an antitussive. b. an expectorant. c. a mucolytic. d. a decongestant.

d

Adverse effects may limit the usefulness of TCAs with some patients. Nursing interventions that could alleviate some of the unpleasant aspects of these adverse effects include a. always administering the drug when the patient has an empty stomach. b. reminding the patient not to void before taking the drug. c. increasing the dose to override the adverse effects. d. taking the major portion of the dose at bedtime to avoid experiencing drowsiness and the unpleasant anticholinergic effects.

d

After teaching a community group about the common cold, the instructor determines that the teaching was successful when the group states which as the cause? a. Bacteria that grow best in the cold b. Allergens in the environment c. Irritation of the delicate mucous membrane d. A number of different viruses

d

Dornase alfa (Pulmozyme), because of its mechanism of action, is reserved for use in a. clearing secretions before diagnostic tests. b. facilitating the removal of secretions postoperatively. c. protecting the liver from acetaminophen toxicity. d. relieving the buildup of secretions in cystic fibrosis.

d

Droxidopa, an antihypotensive drug, should be used a. only with patients who are confined to bed. b. in the treatment of acute shock. c. in patients with known pheochromocytoma. d. to treat orthostatic hypotension in patients whose lives are impaired by the disorder.

d

During an assessment, a client tells the nurse that she "can't stand her mother" and does "whatever she wants me to do" because the client "can't do anything right anyway." The nurse uses this information to determine which item during the client assessment? a. Personal identity b. Role performance c. Body image d. Self-esteem

d

Hypertension is associated with a. loss of vision. b. strokes. c. atherosclerosis. d. all of the above.

d

Leukotriene receptor antagonists act to block production of a component of slow-reacting substance of anaphylaxis. They are most beneficial in treating a. seasonal rhinitis. b. pneumonia. c. COPD. d. asthma.

d

Lung surfactants used therapeutically are a. injected into a developed muscle. b. instilled via a nasogastric tube. c. injected into the umbilical artery. d. instilled into an endotracheal tube properly placed in the baby's lungs.

d

Mr. Jones is the chief executive officer of a large company and has been experiencing acute anxiety attacks. His physical examination was normal, and he was diagnosed with anxiety. Considering his occupation and his need to be alert and present to large groups on a regular basis, which anxiolytic would be the drug of choice for Mr. Jones? a. Phenobarbital b. Diazepam c. Clorazepate d. Buspirone

d

Nitrates are commonly used antianginal drugs that act to a. increase the preload on the heart. b. increase the afterload on the heart. c. dilate coronary vessels to increase the delivery of oxygen through those vessels. d. decrease venous return to the heart, decreasing the myocardial workload.

d

Patients with seasonal rhinitis experience irritation and inflammation of the nasal passages and passages of the upper airways. Treatment for these patients might include a. systemic corticosteroids. b. mucolytic agents. c. an expectorant. d. topical nasal steroids.

d

The activity of the heart depends on both the inherent properties of the cardiac muscle cells and the activity of the autonomic nerves to the heart. Therefore, a. cutting all of the autonomic nerves to the heart produces a decrease in resting heart rate. b. blocking the parasympathetic nerves to the heart decreases the heart rate. c. stimulating the sympathetic nerves to the heart increases the time available to fill the ventricles during diastole. d. the heart rate will increase in cases of dehydration, which will lead to less filling time.

d

The bile acid sequestrants a. are absorbed into the liver. b. take several weeks to show an effect. c. have no associated adverse effects. d. prevent bile salts from being reabsorbed.

d

The low-molecular-weight heparin of choice for preventing DVT after hip replacement therapy is a. heparin. b. betrixaban. c. fondaparinux. d. enoxaparin.

d

The stepped-care approach to the treatment of hypertension includes a. lifestyle modification, including exercise, diet, and decreased smoking and alcohol intake. b. use of a diuretic, beta-blocker, or ACE inhibitor to supplement lifestyle changes. c. a combination of antihypertensive drug classes to achieve desired control. d. all of the above.

d

Venlafaxine (Effexor) is an antidepressant that might be very effective for use in patients who a. are being treated effectively with a SSRI. b. can tolerate multiple side effects. c. are reliable at taking multiple daily dosings. d. have not responded to other antidepressants and would benefit from once-a-day dosing.

d

Which of the following medications is NOT indicated for obsessive-compulsive disorder, depression, and panic disorder? a. citalopram (Celexa) b. paroxetine (Paxil) c. fluvoxamine (Luvox) d. vortioxetine (Brintellix)

d

Which would alert the nurse to suspect that a patient receiving HMG-CoA reductase inhibitors is developing rhabdomyolysis? a. Flatulence and abdominal bloating b. Increased bleeding and bruising c. The development of cataracts and blurred vision d. Muscle pain and weakness

d

Your patient is being treated for depression and is started on a regimen of fluoxetine (Prozac). She calls you 10 days after the drug therapy has started to report that nothing has changed and she wants to try a different drug. You should a. tell her to try sertraline (Zoloft) because some patients respond to one SSRI and not another. b. ask her to try a few days without the drug to see whether there is any difference. c. add an MAOI to her drug regimen to get an increased antidepressant effect. d. encourage her to keep taking the drug as prescribed because it usually takes up to 4 weeks to see the full antidepressant effect.

d

A hospice nurse is critically evaluating various models of grief used for terminally ill clients and their families. Which should the nurse recognize when applying these models to individual client cases? a. There is strong research proving that these models are not useful for many dying clients. b. The Kübler-Ross model is primarily used to describe anticipatory grief. c. The models serve as clear and definitive predictors of grief behaviors. d. No clear timetables exist, nor are there clear-cut stages of grief.

d Although the models of grief are useful in guiding the nursing care of clients who are experiencing loss, there are no clear-cut stages of grief, nor are there exact timetables. Kübler-Ross describes all stages of grief and grieving. None of the models clearly or definitively predict grief behaviors. No research exists that proves these models are not useful.

During a home care visit, an older adult client states to the nurse, "my wife died 3 years ago." Which client action is a possible indicator that the client is experiencing complicated grief? a. The client has an album of photographs of his wife open on the living room table. b. The client indicates that he sends his laundry out to be done because he had never figured out how the washer works. c. The client tells the nurse that his wife was an awful cook and that he has eaten better meals since she died. d. The client shows the nurse his wife's craft room and states that it remains just as she left it before she died

d Leaving the deceased wife's craft room and belongings intact for over 3 years is considered outside the normal limits of the grief process for the elderly. Talking about his deceased wife's good and bad points and showing photographs of her are normal responses to grief. Sending out the laundry to be done is a healthy response to a problem that this client identified.

When providing nursing care the client states, "I drink a small glass of warm water mixed with the juice of one lemon every morning because it helps to heal my body." Which action by the nurse is appropriate when providing care to this client? a. Tell the client that cold water is better metabolized by the body b. Suggest the client delay the water and lemon until after morning medications. c. Instruct the client that lemon juice is really a dose of vitamin C that helps with healing. d. Provide the warm water and juice of a lemon.

d To support the client's beliefs about healing, the nurse should provide the client with the warm water and lemon juice. The nurse should not instruct the client about the benefits of lemon juice being vitamin C. The nurse should also not suggest that cold water be used instead. Asking the client to delay drinking the water and lemon juice will not support the client's spiritual needs.

A nurse is teaching a patient how to use a meter-dosed inhaler for her asthma. Which comments from the patient assure the nurse that the teaching has been effective? Select all that apply. a. "I will be careful not to shake up the canister before using it." b. "I will hold the canister upside down when using it." c. "I will inhale the medication through my nose." d. "I will continue to inhale when the cold propellant is in my throat." e. "I will only inhale one spray with one breath." f. "I will activate the device while continuing to inhale."

d, e, f. Common mistakes that patients make when using MDIs include failing to shake the canister, holding the inhaler upside down, inhaling through the nose rather than the mouth, inhaling too rapidly, stopping the inhalation when the cold propellant is felt in the throat, failing to hold their breath after inhalation, and inhaling two sprays with one breath.

A nurse working in a sleep lab observes the developmental factors that may affect sleep. Which statements accurately describe these variations? Select all that apply. a. REM sleep constitutes much of the sleep cycle of a preschool child. b. By the age of 8 years, most children no longer take naps. c. Sleep needs usually decrease when physical growth peaks. d. Many adolescents do not get enough sleep. e. Total sleep decreases in adults with a decrease in stage IV sleep. f. Sleep is less sound in older adults and stage IV sleep may be absent.

d, e, f. Many adolescents do not get enough sleep due to the stresses of school, activities, and part-time employment causing restless sleep. Total sleep time decreases during adult years, with a decrease in stage IV sleep. Sleep is less sound in older adults, and stage IV sleep is absent or considerably decreased. REM sleep constitutes much of the sleep cycle of a young infant, and by the age of 5 years, most children no longer nap. Sleep needs usually increase when physical growth peaks.

A visiting nurse is performing a family assessment of a young couple caring for their newborn who was diagnosed with cerebral palsy. The nurse notes that the mother's hair and clothing are unkempt and the house is untidy, and the mother states that she is "so busy with the baby that I don't have time to do anything else." What would be the priority intervention for this family? a. Arrange to have the infant removed from the home. b. Inform other members of the family of the situation. c. Increase the number of visits by the visiting nurse. d. Notify the care provider and recommend respite care for the mother.

d. A person providing care at home for a family member for long periods of time often experiences caregiver burden, which may be manifested by chronic fatigue, sleep disorders, and an increased incidence of stress-related illnesses, such as hypertension and heart disease. The nurse should address the issue with the primary care provider and recommend a visit from a social worker or arrange for respite care for the family.

A patient states she feels so isolated from her family and church, and even from God, "in this huge medical center so far from home." A nurse is preparing nursing goals for this patient. Which is the best goal for the patient to relieve her spiritual distress? a. The patient will express satisfaction with the compatibility of her spiritual beliefs and everyday living. b. The patient will identify spiritual beliefs that meet her need for meaning and purpose. c. The patient will express peaceful acceptance of limitations and failings. d. The patient will identify spiritual supports available to her in this medical center.

d. Each of the four options represents an appropriate spiritual goal, but identifying spiritual supports available to this patient in the medical center demonstrates a goal to decrease her sense of isolation.

The Roman Catholic family of a baby who was born with hydroencephalitis requests a baptism for their infant. Why is it imperative that the nurse provides for this baptism to be performed? a. Baptism frequently postpones or prevents death or suffering. b. It is legally required that nurses provide for this care when the family makes this request. c. It is a nursing function to assure the salvation of the baby. d. Not having a Baptism for the baby when desired may increase the family's sorrow and suffering.

d. Failure to ensure that an infant baptism is performed when parents desire it may greatly increase the family's sorrow and suffering, which is an appropriate nursing concern. Whether baptism postpones or prevents death and suffering is a religious belief that is insufficient to bind all nurses. There is no legal requirement regarding baptism, and although some nurses may believe part of their role is to ensure the salvation of the baby, this function would understandably be rejected by many.

A nurse interviews a patient who was abused by her partner and is staying at a shelter with her three children. She tells the nurse, "I'm so worried that my husband will find me and try to make me go back home." Which data would the nurse most appropriately document? a. "Patient displays moderate anxiety related to her situation." b. "Patient manifests panic related to feelings of impending doom." c. "Patient describes severe anxiety related to her situation." d. "Patient expresses fear of her husband."

d. Fear is a feeling of dread in response to a known threat. Anxiety, on the other hand, is a vague, uneasy feeling of discomfort or dread from an often unknown source. Panic causes a person to lose control and experience dread and terror, which can lead to exhaustion and death; that is not the case in this situation.

A patient with COPD is unable to perform personal hygiene without becoming exhausted. What nursing intervention would be appropriate for this patient? a. Assist with bathing and hygiene tasks even if the patient feels capable of performing them alone. b. Teach the patient not to talk about the procedure, just to perform it at the best of his or her ability. c. Teach the patient to take short shallow breaths when performing hygiene measures. d. Group personal care activities into smaller steps, allowing rest periods between activities.

d. For a patient who is too fatigued to complete daily hygiene on his or her own, the nurse should group personal care activities into smaller steps and allow rest periods between the activities. The nurse should assist with bathing and hygiene tasks as needed and only when the patient has difficulty. The nurse should encourage the patient to voice feelings and concerns about self-care deficits, and teach the patient to coordinate diaphragmatic breathing with the activity.

Even though the nurse performs a detailed nursing history in which spirituality is assessed on admission, problems with spiritual distress may not surface until days after admission. What is the probable explanation? a. Patients usually want to conceal information about their spiritual needs. b. Patients are not concerned about spiritual needs until after their spiritual adviser visits. c. Family members and close friends often initiate spiritual concerns. d. Illness increases spiritual concerns, which may be difficult for patients to express in words.

d. Illness may increase spiritual concerns, which many patients find difficult to express. The other options do not correspond to actual experience.

A sophomore in high school has missed a lot of school this year because of leukemia. He said he feels like he is falling behind in everything, and misses "hanging out at the mall" with his friends most of all. For what disturbance in self-concept is this patient at risk? a. Personal Identity Disturbance b. Body Image Disturbance c. Self-Esteem Disturbance d. Altered Role Performance

d. Important roles for this patient are being a student and a friend. His illness is preventing him from doing either of these well. This self-concept disturbance is basically one that concerns role performance.

While assessing a patient in the PACU, a nurse notes increased wound drainage, restlessness, a decreasing blood pressure, and an increase in the pulse rate. The nurse interprets these findings as most likely indicating: a. Thrombophlebitis b. Atelectasis c. Infection d. Hemorrhage

d. Increased wound drainage, restlessness, decreasing blood pressure, and increasing pulse rate are assessment findings that indicate hemorrhage. Thrombophlebitis is an inflammation of a vein associated with thrombus (blood clot) formation. Thrombophlebitis is typically superficial and, in patients without an underlying condition, is often related to IV catheters. Manifestations of atelectasis include decreased lung sounds over the affected area, dyspnea, cyanosis, crackles, restlessness, and apprehension. Signs of infection include elevated white blood count and fever.

A nurse is explaining the rationale for performing leg exercises after surgery. Which reason would the nurse include in the explanation? a. Promote respiratory function b. Maintain functional abilities c. Provide diversional activities d. Increase venous return

d. Leg exercises assist in preventing muscle weakness, promote venous return, and decrease complications related to venous stasis. As a result, the patient has a decreased risk for thrombophlebitis, DVT, and emboli.

A nurse is responsible for preparing patients for surgery in an ambulatory care center. Which technique for reducing anxiety would be most appropriate for these patients? a. Discouraging oververbalization of fears and anxieties b. Focusing on the outcome as opposed to the details of the surgery c. Providing time alone for reflection on personal strengths and weaknesses d. Mutually determining expected outcomes of the care plan

d. Nurses preparing patients for surgery should mutually determine expected outcomes of the care, as well as encourage verbalizations of feelings, perceptions, and fears. The nurse should explain all procedures and sensations likely to be experienced during the procedures, and stay with the patient to promote safety and reduce fear.

A patient whose last name is Goldstein was served a kosher meal ordered from a restaurant on a paper plate because the hospital made no provision for kosher food or dishes. Mr. Goldstein became angry and accused the nurse of insulting him: "I want to eat what everyone else does—and give me decent dishes." Analysis of these data reveals what finding? a. The nurse should have ordered kosher dishes also. b. The staff must have behaved condescendingly or critically. c. Mr. Goldstein is a problem patient and difficult to satisfy. d. Mr. Goldstein was stereotyped and not consulted about his dietary preferences.

d. On the basis of his name alone, the nurse jumped to the premature and false conclusion that this patient would want a kosher diet.

A nurse is explaining pain control methods to a patient undergoing a bowel resection. The patient is interested in the PCA pump and asks the nurse to explain how it works. What would be the nurse's correct response? a. "The pump allows the patient to be completely free of pain during the postoperative period." b. "The pump allows the patient to take unlimited amounts of medication as needed." c. "The pump allows the patient to choose the type of medication given postoperatively." d. "The pump allows the patient to self-administer limited doses of pain medication."

d. PCA infusion pumps allow patients to self-administer doses of pain-relieving medication within health care provider-prescribed time and dose limits. Patients activate the delivery of the medication by pressing a button on a cord connected to the pump or a button directly on the pump.

The family of a patient who has just died asks to be alone with the body and asks for supplies to wash the body. The nurse providing care knows that the mortician usually washes the body. Which response would be most appropriate? a. Inform the family that there is no need for them to wash the body since the mortician typically does this. b. Explain that hospital policy forbids their being alone with the deceased patient and that hospital supplies are to be used only by hospital personnel. c. Give the supplies to the family but maintain a watchful eye to make sure that nothing unusual happens. d. Provide the requested supplies, checking if this request is linked to their religious or cultural customs and asking if there is anything else you can do to help.

d. The family may want to wash the body for personal, religious, or cultural reasons and should be allowed to do so.

A nurse asks a patient who has few descriptors of his self to list facts, traits, or qualities that he would like to be descriptive of himself. The patient quickly lists 25 traits, all of which are characteristic of a successful man. When asked if he knows anyone like this, he replies, "My father; I wish I was like him." What does the discrepancy between the patient's description of himself as he is and as he would like to be indicate? a. Negative self-concept b. Modesty (lack of conceit) c. Body image disturbance d. Low self-esteem

d. The nurse can obtain a quick indication of a patient's self-esteem by using a graphic description of self-esteem as the discrepancy between the "real self" (what we think we really are) and the "ideal self" (what we think we would like to be). The nurse would have the patient plot two points on a line—real self and ideal self (Fig. 41-5). The greater the discrepancy, the lower the self-esteem; the smaller the discrepancy, the higher the self-esteem.

A 36-year-old woman enters the emergency department with severe burns and cuts on her face after an auto accident in a car driven by her fiancé of 3 months. Three weeks later, her fiancé has not yet contacted her. The patient states that she is very busy and she is too tired to have visitors anyway. The patient frequently lies with her eyes closed and head turned away. What do these data suggest? a. There is no disturbance in self-concept. b. This patient has ego strength and high self-esteem but may have a disturbance of body image. c. The area of self-esteem has very low priority at this time and should be ignored until much later. d. It is probable that there are disturbances in self-esteem and body image.

d. The traumatic nature of this patient's injuries, her fiancé's failure to contact her, and her withdrawal response all point to potential problems with both body image and self-esteem. It is not true that self-esteem needs are of low priority.

A nurse providing care of a patient's chest drainage system observes that the chest tube has become separated from the drainage device. What would be the first action that should be taken by the nurse in this situation? a. Notify the health care provider. b. Apply an occlusive dressing on the site. c. Assess the patient for signs of respiratory distress. d. Put on gloves and insert the chest tube in a bottle of sterile saline.

d. When a chest tube becomes separated from the drainage device, the nurse should submerge the end in water, creating a water seal, but allowing air to escape, until a new drainage unit can be attached. This is done instead of clamping to prevent another pneumothorax. Then the nurse should assess vital signs and notify the health care provider.

A nurse is suctioning the nasopharyngeal airway of a patient to maintain a patent airway. For which condition would the nurse anticipate the need for a nasal trumpet? a. The patient vomits during suctioning. b. The secretions appear to be stomach contents. c. The catheter touches an unsterile surface. d. A nosebleed is noted with continued suctioning.

d. When nosebleed (epistaxis) is noted with continued suctioning, the nurse should notify the health care provider and anticipate the need for a nasal trumpet. The nasal trumpet will protect the nasal mucosa from further trauma related to suctioning.

A nurse is providing teaching for a patient scheduled to have same-day surgery. Which teaching method would be most effective in preoperative teaching for ambulatory surgery? a. Lecture b. Discussion c. Audiovisuals d. Written instructions

d. Written instructions are most effective in providing information for same-day surgery.


Set pelajaran terkait

Chapter 18. Disorders of Aging and Cognition

View Set

MAACT Test 2 Relevant Costs and Benefits for Decision Making

View Set

Calcitonin and Parathyroid Hormone

View Set

Chapter 12: Population Growth & Aging

View Set

Fluid, Electrolyte, and Acid-Base Regulation

View Set

Art History: Renaissance - Modern

View Set